Cosmetic Chemical Peel/Skin, Fillers 01-22 Flashcards

1
Q

A 45-year-old woman underwent nonsurgical facial rejuvenation 2 days ago. The procedure included injection of neuromodulators into the forehead and glabella and hyaluronic acid into the tear troughs, nose, cheeks, and lips. The patient reports worsening pain, and discoloration of the nasal tip is noted. Which of the following is the most appropriate next step in management of this complication?

A) Antibiotic therapy
B) Aspirin therapy
C) Hyaluronidase injection
D) Hyperbaric oxygen therapy
E) Warm compress application

A

The correct response is Option C.

This patient has pustulation, erythema, and pain, which indicate intravascular injection and tissue ischemia. Complications with hyaluronic acid are categorized as early (<14 days), late (14 days to 1 year), or delayed (>1 year). Early but rare complications are frequently due to either infection (bacterial or viral) or the much-feared tissue necrosis/embolism. Avoiding these complications with careful technique and knowledge of anatomy is paramount, but understanding proper management of complications is also critical. This patient has signs of tissue necrosis, and the most critical intervention is to inject hyaluronidase (HYAL), an enzyme that catalyzes hyaluronic acid hydrolysis. Hyaluronidase should be injected immediately and liberally and titrated for effect to reverse signs and symptoms such as pallor, livedo reticularis pattern, erythema, and pain. Consider repeating daily as needed. There is no need to attempt to cannulate the vasculature since hyaluronidase will diffuse into the tissue. This product should be readily available to any clinician using hyaluronic acid fillers. Allergic reactions to hyaluronidase can also be noted (especially in patients with history of insect venom allergy).

Warm compresses, aspirin, and hyperbaric oxygen have all been advocated as adjunct measures when there is suspicion of vascular compromise to soft tissue. They are supplemental but not the primary treatment of choice. There are no clinical studies to support hyperbaric oxygen, but animal models suggest it is useful for ischemic injuries.

Empiric antibiotics are appropriate for early acute infection especially if there is obvious abscess or nonfluctuant cellulitis. However, the pustules and erythema in this patient are due to intravascular injection. Antibiotics may be added but should not be the primary or sole treatment plan.

How well did you know this?
1
Not at all
2
3
4
5
Perfectly
2
Q

A 52-year-old woman presents with deepening of the nasolabial and nasojugal folds along with loss of volume of the midface. She is interested in facial fillers to enhance volume in these areas. A hyaluronic acid filler with a higher elastic modulus (G’) value is most appropriate for this patient because which of the following is increased?

A) Injection viscosity
B) Longevity value
C) Product stiffness
D) Swelling factor
E) Tissue dispersion

A

The correct response is Option C.

G’ (elastic modulus) represents the ability of a hyaluronic acid filler to resist forces of deformation and return to its original shape. The higher the G’ value, the stiffer the product and the greater the tissue support that the product is intended to provide. A gel’s ability to expand as it binds water is its swelling factor and is a measure of its hydration.

Swelling factor is usually inversely related to G’, so swelling factor is incorrect. Tissue dispersion is incorrect since a more elastic or stiffer gel (higher G’) will be less likely to spread throughout the tissue when injected. Viscosity, or viscous modulus (G’’), is a measure of a gel’s ability to dissipate energy when shear force is applied to it. Neither viscosity nor longevity are represented as G’, thus making these choices also incorrect.

How well did you know this?
1
Not at all
2
3
4
5
Perfectly
3
Q

A 28-year-old woman comes to see the plastic surgeon because she is unhappy about her “square jaw” and desires a softer shape for her face without surgical intervention. Treatment of her masseter hypertrophy with botulinum toxin was recommended. The patient should be informed that the most likely effect from repeated injections is which of the following?

A) She can be assured that the use of botulinum toxin injection for masseter hypertrophy is within FDA guidelines
B) She should take NSAIDs in anticipation of headaches after botulinum toxin injection
C) She will not have any functional issues with eating and chewing after botulinum toxin injection
D) The volume and bulk of the masseter muscle will be significantly decreased with botulinum toxin injections

A

The correct response is Option D.

A recent study (Shome et al.) demonstrated that after a series of botulinum toxin injections, masseter volume reduction lasted up to 4 years. Therefore, this treatment does not only have short-term effects.

In a review (Peng et al.), 30% of patients reported temporary decrease in mastication force after botulinum injection. Headaches are infrequently reported and happen immediately after injection; they resolve after 2 to 4 days. When patients’ masseter volumes were measured before and after treatment, there was significant reduction in masseter volume (Chang et al.).

Currently, botulinum toxin injection is an off-label use in treatment of masseter hypertrophy.

How well did you know this?
1
Not at all
2
3
4
5
Perfectly
4
Q

A 43-year-old man presents to the office after being treated with botulinum toxin type A for concerning horizontal rhytids in the forehead 3 weeks ago. The patient reports that he is dissatisfied with the results and can barely open his eyes. Physical examination shows ptosis of the left upper eyelid. Apraclonidine 0.5% eye drops are prescribed and the patient reports temporary improvement. Which of the following muscles is most likely the target of the eye drops and involved in the improvement of this patient’s condition?

A) Corrugator supercilia
B) Frontalis
C) Levator palpebrae superioris
D) Orbicularis oculi
E) Superior tarsal (Müller) muscle

A

The correct answer is E.

The superior tarsal muscle is a smooth muscle and also known as Müller’s muscle. An adverse effect of the injection of botulinum toxin type A into the upper third of the face is ptosis, or lid droop, and can occur up to 2 weeks after injections. Ptosis results from migration of the botulinum toxin type A to the levator palpebrae superioris muscle. The levator allows the eyelid to open properly and fully. To avoid ptosis, injections should occur at least 1 cm above the eyebrow.

The superior tarsal muscle originates on the underside of the levator palpebrae superioris and inserts on the superior tarsal plate of the eyelid. The superior tarsal muscle receives its innervation from the sympathetic nervous system. Post-ganglionic sympathetic fibers originate in the superior cervical ganglion, and travel via the internal carotid plexus, where small branches communicate with the oculomotor nerve as it passes through the cavernous sinus. The sympathetic fibers continue to the superior division of the oculomotor nerve, where they enter the superior tarsal muscle on its inferior aspect. The superior tarsal muscle works to keep the upper eyelid raised after the levator palpebrae superioris has raised the upper eyelid. Apraclonidine is an ?2-adrenergic agonist, which causes Müller muscles to contract, quickly elevating the upper eyelid 1 to 3 mm.

In this situation, the frontalis muscle was the target of the botulinum toxin to temporarily alleviate cosmetic concerns of horizontal rhytids in the forehead. The patient was injected too close to the brow likely in an attempt to treat all forehead lines. Eyedrops would not help the frontalis muscle contract and elevate the brow. The levator palpebrae superioris is the primary elevator of the upper eyelid. Ptosis results from migration of the botulinum toxin to the levator palpebrae superioris muscle. This skeletal muscle is innervated by the oculomotor nerve (cranial nerve III) and not susceptible to activation by an alpha adrenergic agonist. The orbicularis oculi muscles is facial muscle responsible for closing the eye. The corrugator supercilii is a small, narrow, pyramidal muscle located at the medial end of the eyebrow, beneath the frontalis and just above the orbicularis oculi muscle. This muscle draws the eyebrow downward and medially, producing the vertical wrinkles of the forehead. This muscle when activated would not elevate the upper eyelid.

References

How well did you know this?
1
Not at all
2
3
4
5
Perfectly
5
Q

A 67-year-old woman presents for consultation to address deep facial rhytides and dyschromias. Medical history includes Fitzpatrick Type II. A chemical peel is planned. For which of the following chemical peeling agents is cardiac monitoring recommended?

A) Glycolic acid
B) Phenol
C) Resorcinol
D) Salicylic acid
E) Trichloroacetic acid

A

The correct response is Option B.

Phenol is absorbed through the skin and into the bloodstream. Most is excreted through the urine. Cardiac dysrhythmias are associated with phenol peels. There are no reported deaths related to phenol toxicity in the literature. As a result, it is recommended that patients who undergo a phenol peel have electrocardiographic monitoring and intravenous access.

The dysrhythmias can include supraventricular tachycardia, premature ventricular contractions, atrial fibrillation, and ventricular fibrillation. Although infrequent, they are associated with higher concentrations of phenol and full-face treatments. It is recommended that the treatment be performed in aesthetic units, allowing for 15 minutes to elapse between the treatment of each unit.

Resorcinol and salicylic acid are components of Jessner’s solution.

Cardiac arrhythmias are not a reported risk with trichloroacetic acid, salicylic acid, glycolic acid or resorcinol.

How well did you know this?
1
Not at all
2
3
4
5
Perfectly
6
Q

A 26-year-old Korean woman reports that she is unhappy about the “square appearance of her jawline” and desires a softer facial shape without surgical intervention. Treatment with botulinum toxin type A is recommended. Which of the following is true as it relates to botulinum for masseteric hypertrophy?

A) Botulinum toxin type A injection is approved by the Food and Drug Administration for the treatment of masseter hypertrophy
B) Effects are invariably short-term
C) Headaches, while infrequently reported, usually are reported to occur several weeks after injection
D) Muscle bulk will not show significant volume reduction on CT scan
E) Temporary decrease in mastication force is the most common effect

A

The correct response is Option E.

A recent study by Shome et al demonstrated that after a series of botulinum toxin injections, masseter volume reduction lasted up to 4 years. Therefore, this treatment’s effects are not only short-term.

In a review by Peng et al, 30% of patients reported temporary decrease in mastication force after botulinum injection. Headaches, while infrequently reported, happen immediately after injection and resolve within 2 to 4 days after injection. When patients’ masseter volumes were measured before and after treatment, there was significant reduction in masseter volume (Chang et al).

Currently, botulinum toxin injection is an off-label use in treatment of masseter hypertrophy. Botulinum toxin is approved by the Food and Drug Administration for use in bladder dysfunction, migraine, glabellar lines, primary axillary hyperhidrosis, blepharospasm, strabismus, cervical dystonia, and upper limb spasticity.

How well did you know this?
1
Not at all
2
3
4
5
Perfectly
7
Q

A 67-year-old woman presents for consultation to address deep facial rhytides and dyschromias. Medical history includes Fitzpatrick Type II. A chemical peel is planned. For which of the following chemical peeling agents is cardiac monitoring recommended?

A) Glycolic acid
B) Phenol
C) Resorcinol
D) Salicylic acid
E) Trichloroacetic acid

A

The correct response is Option B.

Phenol is absorbed through the skin and into the bloodstream. Most is excreted through the urine. Cardiac dysrhythmias are associated with phenol peels. There are no reported deaths related to phenol toxicity in the literature. As a result, it is recommended that patients who undergo a phenol peel have electrocardiographic monitoring and intravenous access.

The dysrhythmias can include supraventricular tachycardia, premature ventricular contractions, atrial fibrillation, and ventricular fibrillation. Although infrequent, they are associated with higher concentrations of phenol and full-face treatments. It is recommended that the treatment be performed in aesthetic units, allowing for 15 minutes to elapse between the treatment of each unit.

Resorcinol and salicylic acid are components of Jessner’s solution.

Cardiac arrhythmias are not a reported risk with trichloroacetic acid, salicylic acid, glycolic acid or resorcinol.

How well did you know this?
1
Not at all
2
3
4
5
Perfectly
8
Q

A 50-year-old woman with Fitzpatrick type II skin and deep perioral rhytides is interested in nonsurgical rejuvenation for her perioral rhytides. Which of the following treatment options is most likely to have the best outcome in a single treatment?

A) Jessner peel
B) 0.4% Croton oil (Hetter) peel
C) 20% glycolic acid peel
D) 20% trichloroacetic acid (TCA) peel
E) 88% phenol peel

A

The correct response is Option B.

The patient is noted to have deep perioral rhytids, and for adequate resolution, a stronger peel is required. Deeper peels, however, will have a longer recovery time.

The glycolic peel is the lightest, followed by the Jessner and trichloroacetic acid (TCA).

While TCA can be used for a perioral peel, it would require a concentration of 35 to 40% to be effective in this scenario. A 20% TCA peel would not give the desired outcome.

In a Hetter peel, the Croton oil is the active peeling agent, not the phenol. Strengths of 0.4% to 0.8% Croton oil are typically used in the perioral area. With this technique, lighter concentrations of Croton oil in phenol are used for more delicate areas, like the lower eyelids or neck, medium concentrations for the cheeks, and heavier concentrations for the perioral area.

Of note, phenol by itself (88% USP), when unoccluded, only gives a light to moderate peel. It is not as strong as the 0.4% Croton oil peel.

Glycolic acid peels are superficial peels and are not effective for deep perioral rhytides.

How well did you know this?
1
Not at all
2
3
4
5
Perfectly
9
Q

A 43-year-old man presents to the office after being treated with botulinum toxin type A for concerning horizontal rhytids in the forehead 3 weeks ago. The patient reports that he is dissatisfied with the results and can barely open his eyes. Physical examination shows ptosis of the left upper eyelid. Apraclonidine 0.5% eye drops are prescribed and the patient reports temporary improvement. Which of the following muscles is most likely the target of the eye drops and involved in the improvement of this patient’s condition?

A) Corrugator supercilia
B) Frontalis
C) Levator palpebrae superioris
D) Orbicularis oculi
E) Superior tarsal (Müller) muscle

A

The superior tarsal muscle is a smooth muscle and also known as Müller’s muscle. An adverse effect of the injection of botulinum toxin type A into the upper third of the face is ptosis, or lid droop, and can occur up to 2 weeks after injections. Ptosis results from migration of the botulinum toxin type A to the levator palpebrae superioris muscle. The levator allows the eyelid to open properly and fully. To avoid ptosis, injections should occur at least 1 cm above the eyebrow.

The superior tarsal muscle originates on the underside of the levator palpebrae superioris and inserts on the superior tarsal plate of the eyelid. The superior tarsal muscle receives its innervation from the sympathetic nervous system. Post-ganglionic sympathetic fibers originate in the superior cervical ganglion, and travel via the internal carotid plexus, where small branches communicate with the oculomotor nerve as it passes through the cavernous sinus. The sympathetic fibers continue to the superior division of the oculomotor nerve, where they enter the superior tarsal muscle on its inferior aspect. The superior tarsal muscle works to keep the upper eyelid raised after the levator palpebrae superioris has raised the upper eyelid. Apraclonidine is an ?2-adrenergic agonist, which causes Müller muscles to contract, quickly elevating the upper eyelid 1 to 3 mm.

In this situation, the frontalis muscle was the target of the botulinum toxin to temporarily alleviate cosmetic concerns of horizontal rhytids in the forehead. The patient was injected too close to the brow likely in an attempt to treat all forehead lines. Eyedrops would not help the frontalis muscle contract and elevate the brow. The levator palpebrae superioris is the primary elevator of the upper eyelid. Ptosis results from migration of the botulinum toxin to the levator palpebrae superioris muscle. This skeletal muscle is innervated by the oculomotor nerve (cranial nerve III) and not susceptible to activation by an alpha adrenergic agonist. The orbicularis oculi muscles is facial muscle responsible for closing the eye. The corrugator supercilii is a small, narrow, pyramidal muscle located at the medial end of the eyebrow, beneath the frontalis and just above the orbicularis oculi muscle. This muscle draws the eyebrow downward and medially, producing the vertical wrinkles of the forehead. This muscle when activated would not elevate the upper eyelid.

How well did you know this?
1
Not at all
2
3
4
5
Perfectly
10
Q

A 25-year-old man presents to the clinic because of a 5-year history of excessive bilateral sweating in the axillae. He states that he uses antiperspirant daily, but despite this, he sweats through his shirts multiple times per day. He finds it embarrassing and distracting at his work. Injection of botulinum toxin type A into each axilla is planned. Which of the following is the most appropriate dosage (per side) of the toxin for this therapy?

A) 15 units
B) 25 units
C) 50 units
D) 100 units
E) 150 units

A

The correct response is Option C.

The patient presents with primary hyperhidrosis. His complaints suggest that he is a 3 or 4 out of 4 on the hyperhidrosis disease severity scale. He has fulfilled criteria for treatment in demonstrating two of the following: duration greater than 6 months, frequency greater than once weekly, bilateral symmetrical sweating, onset before the age of 25, positive family history, and cessation while sleeping. Additionally, he has failed antiperspirant therapy for at least 6 months, as a result, intervention is indicated at this time. The appropriate dosing for axillary botulinum toxin type A is 50 units per side. Palmar injections use 100 units and plantar injections 150 to 250 units per side. Treatment for axillary hyperhidrosis has been shown to have good improvement, with 90% of patients having relief for at least 3 months.

How well did you know this?
1
Not at all
2
3
4
5
Perfectly
11
Q

A 36-year-old woman undergoes treatment of forehead wrinkles. Preoperative examination shows transverse rhytides when raising the eyebrows. Her eyebrows are 1 cm above the supraorbital rim. The central forehead is treated with 24 units of botulinum toxin type A. Two weeks later, the patient is unsatisfied with the appearance of her eyebrows. On examination, the medial eyebrow is located 1 cm above the supraorbital rim. Lateral eyebrows are raised significantly, creating an abnormal arch. Injection of additional botulinum toxin type A into which of the following muscles is the most appropriate treatment at this time?

A) Corrugator supercilia
B) Lateral frontalis
C) Levator palpebrae
D) Orbicularis oculi
E) Procerus

A

The correct response is Option B.

This patient has a “Spock’s eyebrow” deformity which occurs when the eyebrow has an excessive arch—the lateral brow is significantly higher compared the medial brow. This can happen when the medial frontalis is overtreated, resulting in a paradoxical effect on the lateral brow and an unnatural elevation of the lateral brow. Treatment of the deformity is with botulinum toxin type A injection into the lateral frontalis muscle (2 cm above the brow) which will allow the lateral brow to descend.

Facial rhytids are classified as static or dynamic. Botulinum toxin type A injections are useful for treating dynamic rhytids, which occur during muscle contraction. The effects of botulinum toxin type A are usually seen within 3 to 10 days and lasts for 3 to 6 months. The frontalis muscles are brow elevators and are the only elevators in the upper face.

The glabella is a commonly treated area and includes the corrugator supercilia and procerus muscles. Contraction of procerus results in horizontal rhytids and contraction of corrugators result in vertical rhytids. Both are brow depressors.

The orbicularis oculi muscle is targeted for the treatment of “crow’s feet lines” or lateral canthal lines along the lateral orbital rim. The orbital portion of the orbicularis oculi depresses the eyebrows.

The levator palpebrae is a muscle in the upper eyelid and functions to elevate and retract the upper eyelid. Eyelid ptosis can occur with botulinum toxin type A treatment of the glabella if there is diffusion of botulinum toxin type A that effects the levator palpebrae.

How well did you know this?
1
Not at all
2
3
4
5
Perfectly
12
Q

A 50-year-old woman with Fitzpatrick type II skin and deep perioral rhytides is interested in nonsurgical rejuvenation for her perioral rhytides. Which of the following treatment options is most likely to have the best outcome in a single treatment?

A) Jessner peel
B) 0.4% Croton oil (Hetter) peel
C) 20% glycolic acid peel
D) 20% trichloroacetic acid (TCA) peel
E) 88% phenol peel

A

The correct response is Option B.

The patient is noted to have deep perioral rhytids, and for adequate resolution, a stronger peel is required. Deeper peels, however, will have a longer recovery time.

The glycolic peel is the lightest, followed by the Jessner and trichloroacetic acid (TCA).

While TCA can be used for a perioral peel, it would require a concentration of 35 to 40% to be effective in this scenario. A 20% TCA peel would not give the desired outcome.

In a Hetter peel, the Croton oil is the active peeling agent, not the phenol. Strengths of 0.4% to 0.8% Croton oil are typically used in the perioral area. With this technique, lighter concentrations of Croton oil in phenol are used for more delicate areas, like the lower eyelids or neck, medium concentrations for the cheeks, and heavier concentrations for the perioral area.

Of note, phenol by itself (88% USP), when unoccluded, only gives a light to moderate peel. It is not as strong as the 0.4% Croton oil peel.

Glycolic acid peels are superficial peels and are not effective for deep perioral rhytides

How well did you know this?
1
Not at all
2
3
4
5
Perfectly
13
Q

A 36-year-old woman undergoes treatment of forehead wrinkles. Preoperative examination shows transverse rhytides when raising the eyebrows. Her eyebrows are 1 cm above the supraorbital rim. The central forehead is treated with 24 units of botulinum toxin type A. Two weeks later, the patient is unsatisfied with the appearance of her eyebrows. On examination, the medial eyebrow is located 1 cm above the supraorbital rim. Lateral eyebrows are raised significantly, creating an abnormal arch. Injection of additional botulinum toxin type A into which of the following muscles is the most appropriate treatment at this time?

A) Corrugator supercilia
B) Lateral frontalis
C) Levator palpebrae
D) Orbicularis oculi
E) Procerus

A

The correct response is Option B.

This patient has a “Spock’s eyebrow” deformity which occurs when the eyebrow has an excessive arch—the lateral brow is significantly higher compared the medial brow. This can happen when the medial frontalis is overtreated, resulting in a paradoxical effect on the lateral brow and an unnatural elevation of the lateral brow. Treatment of the deformity is with botulinum toxin type A injection into the lateral frontalis muscle (2 cm above the brow) which will allow the lateral brow to descend.

Facial rhytids are classified as static or dynamic. Botulinum toxin type A injections are useful for treating dynamic rhytids, which occur during muscle contraction. The effects of botulinum toxin type A are usually seen within 3 to 10 days and lasts for 3 to 6 months.

The frontalis muscles are brow elevators and are the only elevators in the upper face.

The glabella is a commonly treated area and includes the corrugator supercilia and procerus muscles. Contraction of procerus results in horizontal rhytids and contraction of corrugators result in vertical rhytids. Both are brow depressors.

The orbicularis oculi muscle is targeted for the treatment of “crow’s feet lines” or lateral canthal lines along the lateral orbital rim. The orbital portion of the orbicularis oculi depresses the eyebrows.

The levator palpebrae is a muscle in the upper eyelid and functions to elevate and retract the upper eyelid. Eyelid ptosis can occur with botulinum toxin type A treatment of the glabella if there is diffusion of botulinum toxin type A that effects the levator palpebrae.

How well did you know this?
1
Not at all
2
3
4
5
Perfectly
14
Q

Which of the following disorders is most likely to be successfully treated with botulinum toxin type A?

A) Bruxism
B) Cervical paroxysmal dyskinesia
C) Hemifacial dyskinesia
D) Tardive dyskinesia
E) Vertical maxillary excess

A

The correct response is Option C.

Botulinum toxin type A has good evidence demonstrating efficacy for the treatment of blepharospasms, hemifacial dyskinesia, cervical dystonia, post-stroke upper limb spasticity (not from paroxysmal dyskinesia—see below).

Bruxism is clenching/grinding of teeth and jaw and can occur either awake or asleep. Treatment includes splint therapy and pharmacotherapy. There have been published reports using botulinum toxin type A. However, while the amplitude of muscle contraction was decreased, it did not decrease the rhythm or number of bruxism episodes. At this time, there is no evidence that it is an effective treatment.

Tardive dyskinesia develops as a result of adverse effect of anti-psychotic medication. Botulinum toxin type A, which consists of VMAT2 (vesicular monoamine transporter 2) inhibitor such as deutetrabenazine or valbenazine, is not indicated in its treatment.

Paroxysmal dyskinesia is considered an immune-related movement disorder. It mainly affects the limbs but can also affect cervical muscles. Treatment includes systemic antibiotics, corticosteroids, and immune therapy such as rituximab. Botulinum toxin type A is not indicated.

Vertical maxillary excess is not treated with botulinum toxin type A.

How well did you know this?
1
Not at all
2
3
4
5
Perfectly
15
Q

Which of the following anatomic landmarks should be used during percutaneous injection of neurotoxin to treat benign masseteric hypertrophy?

A) Between the earlobe-to-oral commissure line and the inferior border of the mandible
B) Between the maxillary and mandibular occlusal planes
C) Between the tragus-to-infraorbital rim line and the cervicomental line
D) Between the tragus-to-mid upper lip line and the root of the helix to subnasale line
E) Between the zygomatic arch-to-mid upper lip line and the Frankfort horizontal

A

The correct response is Option A.

Benign masseteric hypertrophy can be treated by botulinum toxin type A or other neurotoxins. The masseter originates from the body of the zygoma, and inserts into the inferior border of the mandible, and therefore the inferior limit is the inferior border of the mandible. Below the transverse line from the earlobe to the corner of the mouth is a safe zone, as there are no important anatomic structures inferior to this line, and yet the majority of the masseter muscle hypertrophy will be in this region. The other choices contain other anatomic landmarks that are not related to neurotoxin injection for benign masseter hypertrophy.

How well did you know this?
1
Not at all
2
3
4
5
Perfectly
16
Q

Chemical peel neutralization using 1% sodium bicarbonate would be recommended with which of the following peeling agents?

A) Glycolic acid
B) Jessner solution
C) Phenol-croton oil
D) Salicylic acid
E) Trichloroacetic acid (TCA)

A

The correct response is Option A.

The requirement for neutralization is specific to each peel and must be thoroughly understood before application. For instance, the trichloroacetic acid peel approach is a dynamic process that depends largely on clinical judgment and experience to assess the extent of frosting relative to peel depth in an effort to optimize results and minimize complications. Furthermore, peel neutralization is typically carried out with a basic solution such as 1% sodium bicarbonate, and is generally required only for specific acids, such as glycolic acid, whereas phenol-croton oil peels and Jessner solution cannot be neutralized. Salicylic acid does not need to be neutralized. TCA penetrates deep to the dermal-epidermal junction and is counteracted by dilution with saline. TCA cannot be neutralized at the end of treatment because it has penetrated too deeply.

How well did you know this?
1
Not at all
2
3
4
5
Perfectly
17
Q

A 53-year-old woman is evaluated because of horizontal rhytides at the root of the nose and between her eyebrows. Treatment will most specifically target which of the following muscles?

A) Corrugator supercilii
B) Depressor septi nasi
C) Frontalis
D) Orbicularis oculi
E) Procerus

A

The correct response is Option E.

The procerus stretches from the nasal bones to the dermis of the glabella, and contraction promotes horizontal rhytides in the glabellar region. The corrugator supercilii is more likely to promote vertical rhytides in the glabella. The orbicularis oculi serves as a sphincter around the eye and may also contribute to vertical glabellar rhytides. The frontalis is responsible for the horizontal forehead rhytides above the eyebrows. The depressor septi nasi does not impact forehead wrinkles.

How well did you know this?
1
Not at all
2
3
4
5
Perfectly
18
Q

A 35-year-old woman comes to the office for treatment of glabellar rhytides with botulinum toxin type A. A total of 20 units is used for the treatment, with 10 units injected into the region of each corrugator muscle. Ten days later, the patient returns to the office because of ptosis of the left upper eyelid. To help decrease the eyelid ptosis, an alpha-adrenergic agonist eye drop can be used to stimulate which of the following muscles?

A) Dilator pupillae
B) Frontalis
C) Levator palpebrae superioris
D) Müller
E) Orbicularis oculi

A

The correct response is Option D.

Eyelid ptosis is the result of an inadvertent effect of the botulinum toxin upon the levator palpebrae superioris muscle. This can occur during treatment of the glabellar region if the injection is performed within/below the orbital rim.

The ptosis that results from the weakening of the levator palpebrae superioris can be countered by the use of alpha-adrenergic eyedrops such as apraclonidine (Iopidine) or phenylephrine. These eyedrops will stimulate the Müller muscle, which is an accessory eyelid elevator that is located deep to the levator palpebrae superioris between the levator muscle and the conjunctiva of the upper eyelid. This can help improve the ptosis but not likely adequately resolve the problem until the effect of botulinum toxin type A has worn off.

The levator, frontalis, and orbicularis will not be affected by the use of the eyedrops. The iris will dilate because of the effects of the eyedrops. This however, will have no effect on the ptotic position of the eyelid.

How well did you know this?
1
Not at all
2
3
4
5
Perfectly
19
Q

A 42-year-old woman comes to the office for consultation on nonsurgical options for treatment of prominent nasolabial folds. The consultation covers injectable filler options. Compared with other types of injectable fillers, hyaluronic acid is distinguished by which of the following characteristics?

A) Can be enzymatically reversed
B) Cannot be combined with lidocaine
C) Contains methylmethacrylate
D) Is indicated for subperiosteal placement
E) Is radiopaque on x-ray study

A

The correct response is Option A.

Hyaluronic acid (HA) fillers are among several agents currently available in the United States. HA is a naturally occurring polysaccharide derived from bacterial fermentation that exhibits no species or tissue specificity. Therefore, serial retreatment is unlikely to induce allergic reactions. Other types of filler products include finely ground methylmethacrylate, calcium hydroxyapatite and poly-L-lactic acid. Although complications may arise with any type of filler, intravascular injection/embolization can occur, and it requires urgent management to preclude tissue necrosis or, in the case of retinal artery occlusion, vision loss. Only HA can be dissolved with an injection of an enzyme, hyaluronidase. It is not radiopaque as is calcium hydroxyapatite. It may appear gray (Tyndall effect) if placed too near the surface. While certain formulations of HA can be injected into the subcutaneous plane, neither it nor any of the others are approved for injection into the subperiosteal plane. Lidocaine is mixed into many HA products to diminish injection site pain. More than 1 mL can be injected per patient in the same setting. Operator technique is most important to maximize safety when using any type of filler. Minor complications of fillers include hypersensitivity, acute infection, malposition of filler, and inflammatory nodules or granulomas.

How well did you know this?
1
Not at all
2
3
4
5
Perfectly
20
Q

A 46-year-old woman seeks to improve her facial appearance with soft-tissue fillers. Her rejuvenation plan includes administration of a highly cross-linked hyaluronic acid injectable filler to the mid face to restore volume loss. To properly add volume to and rejuvenate this area, the filler should be injected into which of the following soft-tissue layers?

A) Intradermal
B) Preperiosteal
C) Subdermal
D) Subperiosteal
E) Throughout multiple layers of soft tissue from deep to superficial

A

The correct response is Option B.

Restoration of mid face volume loss is a highly effective maneuver that can be performed utilizing fat grafting or off-the-shelf injectable fillers. The most commonly used hyaluronic acid-based injectable fillers utilized for mid face rejuvenation and volume restoration include those that are highly cross-linked, thus increasing the stability, density, cohesivity, and longevity of the filler. The process of cross-linking hyaluronic acid results in larger, more stable molecules that have biocompatibility and viscoelastic properties similar to those of fat. Ideally, these highly dense fillers should be placed at the preperiosteal level to optimize results and minimize potential complications, such as intravascular placement of filler or visible

How well did you know this?
1
Not at all
2
3
4
5
Perfectly
21
Q

A patient with facial hyperkinesia comes to the office for treatment with botulinum toxin type A for temporary improvement in the appearance of moderate to severe glabellar facial lines. How many units of botulinum toxin type A should be administered to this patient, according to the Food and Drug Administration?

A) 1
B) 10
C) 20
D) 50
E) 100

A

The correct response is Option C.

Two phase 3 randomized, multi-center, double-blind, placebo-controlled studies of identical design were conducted to evaluate botulinum toxin type A prior to FDA approval. The injection volume was 0.1 mL/injection site, for a dose/injection site in the active treatment groups of 4 units. Subjects were injected intramuscularly in five sites—1 in the procerus muscle and 2 in each corrugator supercilii muscle—for a total dose in the active treatment groups of 20 units.

Botulinum toxin type A blocks neuomuscular transmission by binding to acceptor sites on motor nerve terminals, entering the nerve terminals, and inhibiting the release of acetylcholine. One unit corresponds to the calculated median intraperitoneal lethal dose (LD50) in mice. Each vial of botulinum toxin type A contains either 100 units of Clostridium botulinum type A neurotoxin complex, 0.5 mg of albumin (human), and 0.9 mg of sodium chloride, or 50 units of C. botulinum type A neurotoxin complex, 0.25 mg of albumin (human), and 0.45 mg of sodium chloride in a sterile, vacuum-dried form without a preservative.

How well did you know this?
1
Not at all
2
3
4
5
Perfectly
22
Q

A 52-year-old woman comes to the office to receive botulinum toxin type A injections to the corrugator and procerus. She returns to the office 1 week later because she is upset that her eyelids on both sides are droopy. Physical examination shows bilateral ptosis. Which of the following is the most appropriate treatment to improve this patient’s condition until the effects of the botulinum toxin type A subside?

A) Apraclonidine
B) Artificial tears
C) Botulinum toxin type A to the lateral orbicularis oculi
D) Ophthalmic tobramycin and dexamethasone
E) Tetracaine

A

The correct response is Option A.

This patient has developed true eyelid ptosis from her botulinum toxin type A treatments coming into contact and affecting the levator palpebrae superioris muscle within the eyelid. Although the effects of botulinum toxin type A on any muscle are irreversible with medications, attempts to lessen the severity of the ptosis have been made with the use of eyedrops to stimulate the Müller muscle, which is located deep to the levator. Apraclonidine is an alpha-adrenergic agonist and as such stimulates the Müller muscle to contract. This contraction may elevate the eyelid 1 to 3 mm and lessen the amount of ptosis to varying degrees in order to make the overall appearance of the eyelids more tolerable to the patient until the effects of botulinum toxin type A wear off on their own and levator function naturally returns. The most common dose of apraclonidine is 1 to 2 drops three times daily until ptosis resolves.

Tetracaine is a commonly used numbing agent for the corneal surface that enhances the comfort of using corneal protectors for periorbital surgery. Tobradex eyedrops are a combination of tobramycin and dexamethasone used for treatment of infection and/or its anti-inflammatory effect in the periorbital region. It has no effect on eyelid ptosis. Artificial tears are lubricating drops and have no effect on muscular action.

How well did you know this?
1
Not at all
2
3
4
5
Perfectly
23
Q

Accidental injection of hyaluronic acid filler into which of the following areas is most likely to cause blindness by retrograde occlusion of the central retinal artery?

A) Cheek
B) Geniomandibular groove
C) Lateral lip commissure
D) Nasal dorsum
E) Nasolabial fold

A

The correct response is Option D.

The dorsal nasal artery is a distal continuation of the ophthalmic artery from the internal carotid artery. During nasal dorsum augmentation, accidental injection of filler into this artery under pressure can push the filler retrograde into the ophthalmic artery. When the pressure is returned to normal, the filler then can flow distally occluding the retinal artery and causing blindness. Intravascular injection of fillers into the angular artery of the nasolabial line can also cause blindness, although this would more commonly result in skin mottling and necrosis of the nasal tip skin.

The cheek area overlying the malar bone, with few deep vessels, is a relatively safe place for injectables.

Intravascular injection of the geniomandibular groove and lateral lip commissure is more commonly associated with vascular compromise of the lips and chin.

How well did you know this?
1
Not at all
2
3
4
5
Perfectly
24
Q

A 50-year-old woman receives an injection of 0.5 mL of hyaluronic acid filler into each nasolabial fold. She returns to the office 30 minutes later because of pain and mottled skin discoloration of the nasal tip and right ala nasi. Which of the following is the most appropriate next step in management?

A) Doppler ultrasonography
B) Hyaluronidase injection
C) Lidocaine injection
D) Massage
E) Prostaglandin E1 injection

A

The correct response is Option B.

This patient presents with signs and symptoms consistent with an intra-arterial injection of hyaluronic acid (HA), which requires emergency treatment to restore circulation.

Accidental intra-arterial injection of fillers leading to arterial compromise is a rare occurrence. When it occurs, devastating outcomes include blindness, stroke, skin necrosis, and permanent scarring. The presenting signs and symptoms may include pain, skin blanching, mottled skin discoloration, and slow capillary refill. The mainstay of treatment for intra-arterial injection of HA products is local injection of hyaluronidase into the site of injection and the local area of skin mottling. Hyaluronidase is an enzyme that catalyzes HA hydrolysis.

Other treatments include massaging the area in order to promote distribution of hyaluronidase, topical nitropaste, aspirin, and warm compresses. Secondary treatments that can be considered after hyaluronidase injection include hyperbaric oxygen, papavarin, prostaglandin E1, heparin, and lidocaine. Radiologic evaluation with Magnetic resonance angiogram (MRA) or Doppler ultrasound would only delay treatment. Massage alone will not benefit this patient.

Recommendations for risk reduction include the following: using large-bore blunt cannulas (27 gauge and larger), injecting less than 0.1-mL bolus in any single injection site, avoiding high-pressure injections, awareness of likely position of named vessels in the treatment area, using local anesthesia with epinephrine, and caution with deep injections around the radix, lateral nasal wall, and periorbital area.

How well did you know this?
1
Not at all
2
3
4
5
Perfectly
25
Q

Which of the following is the most common histologic effect of skin treatment with tretinoin?

A) Decrease in angiogenesis
B) Decrease in mucin
C) Increase in collagen
D) Increase in melanin
E) Thinning of the epidermis

A

The correct response is Option C.

Retinoids pass through the cell wall via nonreceptor-mediated endocytosis, are carried to the nucleus on cellular retinoic acid-binding proteins (CRABP, CRABP-II), and exert their effect through binding to retinoic acid receptors and retinoid X receptors. The retinoid-receptor complex binds to the promoter gene in the region of the retinoid response elements, resulting in production of proteins responsible for effects we see histologically and grossly.

In sun-damaged skin, the major findings histologically are reduced collagen quantity and dermal collagen disorder. Retinoids have come to be a mainstay in the treatment of photodamaged skin due to their ability to repair this damage. The effects noted histologically of retinoids on photodamaged skin include increased quantity of collagen (types I, III, and VII), greater organization of the collagen within the dermis, improved organization of elastic tissue, epidermal hyperplasia, increased mucin deposition (epidermal and dermal), and decreased melanin, among others. These histologic changes translate into improvement in rhytides, smoother skin, and correction of dyschromia.

Thinning of the epidermis is incorrect because retinoids result in epidermal hyperplasia.

Decrease in mucin deposition is incorrect because retinoids result in increased mucin deposition.

Decrease in angiogenesis is incorrect because retinoids result in increased angiogenesis in the skin.

Increase in melanin is incorrect because retinoids result in a decrease in melanin content of the skin.

How well did you know this?
1
Not at all
2
3
4
5
Perfectly
26
Q

A 45-year-old woman comes to the office because of deep rhytides caused by photoaging. Topical application of 35% trichloroacetic acid in combination with Jessner solution is planned. Which of the following best describes the clinical endpoint during application of this chemical peel?

A) Dark firm eschar
B) Grey hue
C) Hypopigmentation
D) Transparent frost with a pink background
E) Uniform deep white frost

A

The correct response is Option E.

The deep white frost indicates the endpoint for the depth of skin penetration with a deep rhytid chemical peel such as the combination Jessner/35% trichloroacetic acid solution. This indicates that the peel has penetrated into the upper reticular dermis. There is no pink hue because at this level there is vasospasm of the capillaries in the papillary dermis. This depth of penetration is for moderate and deep rhytides. This is a transient phenomenon. Capillary refill should return within 20 to 40 minutes.

A transparent frost with a pink background is the endpoint for a superficial peel, such as that done with a trichloroacetic solution (<30%) alone. This frost is due to the coagulation of proteins in the dermis and epidermis. The pink hue emanates from blood vessels that remain intact in the papillary dermis.

A grey hue indicates a deeper penetration with destruction to the dermis. This level of penetration could lead to abnormal healing, scarring, pigmentation, and texturing changes. Hypopigmentation and superficial epidermolysis are potential complications of a chemical peel and not clinical endpoints to determine depth of penetration.

How well did you know this?
1
Not at all
2
3
4
5
Perfectly
27
Q

Which of the following best describes the mechanism of inhibition of muscle contraction by botulinum toxin type A?

A) Binds to acetylcholine in presynaptic nerves, degrading acetylcholine
B) Binds to acetylcholine in postsynaptic nerves, degrading acetylcholine
C) Binds to acetylcholine in postsynaptic nerves, preventing acetylcholine interaction with nicotinic receptors
D) Binds to nicotinic receptor sites on the neuromuscular junction, blocking acetylcholine stimulation
E) Binds to receptor sites in presynaptic nerve terminals, blocking acetycholine release

A

The correct response is Option E.

Botulinum toxin type A works by binding to receptor sites in presynaptic nerves to prevent the release of acetylcholine into the synapse of neuromuscular junctions.

How well did you know this?
1
Not at all
2
3
4
5
Perfectly
28
Q

A 52-year-old woman seeks treatment for signs of facial aging. She reports diffuse fine rhytides caused by a long history of sun exposure and smoking. Which of the following is most likely to result in the greatest long-term increase in dermal collagen content?

A) Deep-plane rhytidectomy
B) Fat grafting
C) Hyaluronic acid filler injection
D) Tretinoin therapy
E) Trichloroacetic acid peel

A

The correct response is Option D.

While all of the listed treatments are effective and employed commonly for facial aging, only tretinoin (Retin-A) has been found to have effectiveness in long-term collagen production. Retinoids are vitamin A derivatives that have profound effects on the skin. Both increases in dermal collagen production and decreases in degradation are seen over 6 to 12 months of treatment. The active form is tretinoin, a metabolite of vitamin A. Within 3 months of starting treatment, improvements in skin softness, texture, and reductions in fine rhytides and dyschromia are apparent. Treatment is typically a 0.05 to 0.01% topical cream applied nightly. Peeling and redness are common, but treatment tolerance improves with time. Early treatment reactions can be treated with decreased product concentration, longer treatment intervals, and topical hydrocortisones.

Fat grafting is a very effective volume replacement and deeper rhytides treatment. While there are some suggestions of increased vascularity and health of overlying skin, effects are secondary and not as profound as tretinoin. Rhytidectomy (facelifting) results in physical skin tightening by removal, along with deeper tissue (SMAS) repositioning. Skin texture and collagen effects are relatively minor. Hyaluronic acid fillers are also effective rhytide and volume treatments, but have little or no effect on collagen. TCA peels effectively treat the epidermis and superficial dermis and have minor collagen stimulation effects through the natural wound-healing process. While surface appearance effects can be dramatic, the amount of collagen stimulation is far less than tretinoin. Laser resurfacing can result in more dermal injury and resultant collagen production than chemical peels, although hypopigmentation and prolonged recovery are disadvantages. Pre-treatment with tretinoin prior to peels and laser resurfacing can increase the depth of treatment and, some hypothesize, improvements in healing and recovery time.

How well did you know this?
1
Not at all
2
3
4
5
Perfectly
29
Q

A 38-year-old woman seeks cosmetic enhancement of the nasolabial area to decrease deep folds. She has had good results from injection of hyaluronic acid–based soft-tissue fillers in the past but now desires a longer lasting result. Calcium hydroxylapatite is chosen based on this request. On follow-up examination 2 weeks later, white nodules are noted along the nasolabial folds. Which of the following is the LEAST effective treatment option to address the white nodules?

A) Direct excision of the filler
B) Injection of a corticosteroid
C) Massage of the folds
D) Needle disruption and unroofing of the lumps

A

The correct response is Option B.

Calcium hydroxylapatite is a semipermanent material that can be injected as a soft-tissue filler and lasts 1 to 2 years, which is longer than the 4 to 12 months that hyaluronic acid-based fillers last. Safe injection of this material includes prevention of overcorrection, prevention of clumping of filler due to bolus injections, injection in a subdermal plane, and postinjection massage. If nodules form (which usually occur in areas of thin soft-tissue coverage such as the eyelids, lips, and nasolabial region), there are multiple described effective treatments, which include direct excision, observation to allow for the product to resorb, and needle disruption and unroofing. However, whereas lumps caused by poly-L-lactic acid or polymethyl methacrylate respond well to intralesional steroids, these are not as effective in treating lumps caused by calcium hydroxylapatite.

How well did you know this?
1
Not at all
2
3
4
5
Perfectly
30
Q

A 23-year-old man is evaluated 1 day after undergoing a chemical peel to the face, entire back, arms, forearms, and hands, in a nonmedical setting. The patient reports nausea, disorientation, and ringing of the ears. Which of the following chemical peels was most likely used on this patient?

A) Glycolic acid
B) Resorcinol
C) Salicylic acid
D) Solid carbon dioxide slush
E) Trichloroacetic acid

A

The correct response is Option C.

This patient is presenting with symptoms of salicylism or salicylic acid toxicity, a rare side effect of salicylic acid peels. Symptoms can include: rapid breathing, tinnitus, hearing loss, dizziness, abdominal cramps and central nervous system reactions. It is more likely to occur when large surface areas are peeled. It has been reported with 20% salicylic acid applied to 50% of the body surface, and lesser areas when stronger concentrations are used. Therefore, care should be taken when treating skin conditions that cover large surface areas, such as acne or psoriasis, with this peel. In general, however, salicylic acid peels are safe when used in more modestly sized areas (less than 20% TBSA) and have minimal complications. They can be used in darker skin types (IV-VI) successfully.

Salicylism has also been reported when large areas are peeled with Jessner’s solution.

The other listed peels do not exhibit this type of toxic reaction.

How well did you know this?
1
Not at all
2
3
4
5
Perfectly
31
Q

An otherwise healthy 46-year-old woman undergoes botulinum toxin type A (Botox) treatment for severe glabellar lines. Twelve units of Botox is administered into each corrugator muscle. Eight days later, the patient comes to the office because of ptosis of the right eyelid. Which of the following muscles is most likely inadvertently affected in this patient?

A) Frontalis
B) Levator palpebrae superioris
C) Müller muscle
D) Orbicular muscle of the eye
E) Procerus

A

The correct response is Option B.

Ptosis following botulinum toxin type A (Botox) treatment to the glabellar area is most commonly associated with inadvertent exposure of the levator palpebrae superioris muscle to Botox. This muscle is the primary upper eyelid elevator and is innervated by the third cranial nerve. The muscle portion arises from the greater wing of the sphenoid and is typically 40 mm long. The tendinous distal portion is 14 to 20 mm long and is termed the levator aponeurosis. The transition from the muscular to the tendinous portion occurs in the region of Whitnall’s ligament, a condensation of the superior sheath of the levator muscle.

Ptosis from Botox treatments most commonly occurs when the Botox spreads outside the intended target muscle. This is most commonly a technical error on the part of the injector by not staying high enough within the corrugator muscle and above the orbital rim. Ptosis from Botox injections cannot be reversed; however, the condition does completely resolve when the Botox effect wears off after several (3 to 4) months. Interval treatment to help improve, but not definitively treat, the ptosis consists of alpha-adrenergic eyedrops such as Iodipine or phenylephrine ophthalmic preparations, which cause stimulation of Müller’s muscle to help improve the condition somewhat, but do not adequately resolve the ptosis. Müller’s muscle is an accessory eyelid elevator and lies deep to the levator. It is innervated by the sympathetic nervous system. Contraction of this muscle (such as with pharmacologic stimulation) contributes about 2 mm to lid retraction.

How well did you know this?
1
Not at all
2
3
4
5
Perfectly
32
Q

An otherwise healthy 54-year-old woman with Fitzpatrick Type II skin undergoes full-face carbon dioxide laser resurfacing. She received acyclovir for 3 days before the procedure. She is treated with a closed dressing regimen. On postoperative day 2, the patient has onset of facial pain and pruritus. Physical examination shows marked diffuse erythema and edema. Which of the following is the most likely diagnosis?

A) Allergic reaction
B) Bacterial infection
C) Fungal infection
D) Herpes simplex virus
E) Normal healing

A

The correct response is Option E.

Resurfacing causes complete ablation of the epidermis and superficial papillary dermis with thermal injury and coagulation through the papillary dermis. Wound healing occurs by re-epithelialization from the dermal appendages (hair follicles and sebaceous glands) and is complete within 7 to 10 days. The thermally damaged dermal layers are repaired by the stimulation of fibroblasts. This coagulated tissue is replaced by new bundles of tight collagen, a process that continues for up to 6 months. Thus, the result tends to improve over time.

During the initial period of healing and during the re-epithelialization, there is considerable edema and exudation of proteinaceous material, resulting in redness and crusting. Erythema is most intense during the first month after treatment but may persist for 6 months or more.

In addition to intense erythema, other expected adverse effects occurring in virtually all patients during the first postoperative week include marked edema, pain, and pruritus.

Allergic reactions, or contact dermatitis, occur most often in open postoperative dressing regimens where the patient has used topical products that contain irritants. Signs and symptoms suggestive of an allergic contact dermatitis include diffuse and intense facial erythema and/or pruritus.

Bacterial and fungal infections are often the result of prolonged (greater than 48 hours) wound occlusion in the postoperative period. Although the risk of bacterial infection is increased in the closed technique compared with the open, in this case, the dressing was changed at day 1 and it is therefore unlikely. Additional findings on examination suggestive of infection include lesions with skin invasion, focal areas of increased erythema, discoloration, purulent rather than serous discharge, and ulceration. The patient described did not exhibit any of these findings. The most common bacterial pathogen is Staphylococcus aureus, and the most common fungal pathogen is Candida.

Despite adequate antiviral prophylaxis, 2 to 7% of laser-treated patients have been shown to develop herpes simplex virus reactivation. An outbreak on laser-treated skin may have symptoms such as superficial erosions and irregular redness. These findings are not present in this case.

How well did you know this?
1
Not at all
2
3
4
5
Perfectly
33
Q

A 38-year-old woman receives an injection of hyaluronic acid gel fillers to improve the appearance of her nasolabial folds. The evening after she received the injection, the patient calls the answering service and reports to the surgeon that, several hours after the injection, she developed skin “irritation” on the left side of the nose with skin discoloration, swelling, and numbness. Which of the following is the most appropriate next step in management?

A) Evaluate the patient in person
B) Initiate treatment with an oral antihistamine
C) Initiate treatment with an oral benzodiazepine
D) Tell the patient to immediately apply ice
E) Reassurance

A

The correct response is Option A.

The most severe and feared early occurring complication of soft-tissue filler agents is tissue necrosis, caused by interruption of the vascular supply to the area by either direct injury of the vessel, compression of the area around the vessel, or obstruction of the vessel by the filler material. It is a rare event, and although more commonly reported in the glabellar region, it has been reported following injection of the nasolabial fold area with hyaluronic acid gel and calcium hydroxylapatite (Radiesse) filler products, causing alar necrosis.

Treatment options for impending necrosis are based on those recommended for the treatment of the glabella and remain anecdotal. Typically, if noted immediately, injections are halted, warm compresses are applied, and nitroglycerin paste is used for local vasodilatation. Immediate use of hyaluronidase to the injection site is also recommended. The use of hyperbaric oxygen is controversial.

Recognition of the possible problem is essential, so that early intervention can minimize tissue necrosis and subsequent deformity. Ice would potentially worsen the already compromised blood flow to the area and is not recommended. Benzodiazepines and antihistamines do not treat the underlying problem, which is tissue ischemia.

How well did you know this?
1
Not at all
2
3
4
5
Perfectly
34
Q

A 36-year-old woman with Fitzpatrick Type II skin is evaluated because of melasma that is refractory to hydroquinone therapy. A biopsy is performed, and examination of the specimen confirms the diagnosis of mixed melasma extending to the upper reticular dermis. Administration of which of the following is most appropriate to treat this area?

A) Glycolic acid 50 to 70%
B) Jessner solution
C) Salicylic acid 20 to 30%
D) Tretinoin
E) Trichloroacetic acid 35 to 50%

A

The correct response is Option E.

Chemical peeling causes controlled destruction of parts of the epidermis and/or dermis, followed by regeneration of new dermal and epidermal tissues. In a controlled manner, a chemical peel induces injury at a specific depth of the skin. Peels are categorized as superficial, medium depth, or deep, depending on the level of injury. Superficial peels cause necrosis of the epidermis only. Medium-depth peels create a wound through the epidermis into the level of the upper reticular dermis. Deep peels penetrate to the mid reticular dermis.

Salicylic acid 20 to 30% would cause injury to the stratum corneum and possibly the stratum granulosum with exfoliation. The depth is less than 100 µm, which is classified as superficial-very light. Both glycolic acid 50 to 70% and the Jessner solution penetrate to a depth of 100 µm, which is considered superficial-light. These agents cause necrosis of the entire epidermis down to the basal layer and stimulate regeneration of new epithelium. A medium-depth peel extends 200 µm, penetrates through the epidermis and papillary dermis to the upper reticular dermis, and results in increased collagen production. Trichloroacetic acid solution 35 to 50% would penetrate to medium-depth. A deep peel penetrates to deeper than 400 µm and causes necrosis to part or all of the mid reticular dermis.

Careful preprocedure evaluation is imperative in choosing the appropriate peel for each patient. Fitzpatrick skin type must be assessed to determine the risk of post-peel complications. If a patient has had a recent medium or deep peel within the past 3 months, facial surgery with extensive undermining or isotretinoin therapy within the past 6 months, or a history of keloid scarring, then care must be taken when selecting medium-depth or deep peels because the risk of hyperpigmentation and/or permanent scarring is increased.

Patients with mixed and dermal melasma are often difficult to treat because of the deeper pigment. A test spot is helpful in determining the patient’s tolerance for the peel when there is concern about the potential adverse effects.

Tretinoin 0.01% causes increased turnover of follicular epithelial cells and helps prevent collagen loss. It is not indicated for melasma.

How well did you know this?
1
Not at all
2
3
4
5
Perfectly
35
Q

Treatment with poly-L-lactic acid (Sculptra) is most likely to correct the soft-tissue facial deformity associated with which of the following conditions?

A) Discoid lupus
B) Progressive hemifacial atrophy
C) Scleroderma
D) Secondary effects of HIV treatment

A

The correct response is Option D.

Poly-L-lactic acid, marketed as Sculptra, has been utilized for the treatment of HIV retroviral drug–related lipoatrophy since 1999. The images shown illustrate such a patient.

Since its initial use, the indications for utilization have broadened and include most etiologies of lipoatrophy with its second most common use for age-related changes. In patients with age-related facial changes, it has been useful for mid face rejuvenation and temporal hollowing. Similar to age-related changes is weight loss that, when mild, will also respond to poly-L-lactic acid but when severe, will require a rhytidectomy. Lipoatrophy secondary to discoid lupus has been reported to respond best to fat grafting. The least likely to respond to poly-L-lactic acid is progressive hemifacial atrophy due to the severity of the soft-tissue deformity and will usually require soft-tissue augmentation with a free flap in severe cases and fat grafting in mild cases.

How well did you know this?
1
Not at all
2
3
4
5
Perfectly
36
Q

Which of the following is the only cosmetic indication for which onabotulinumtoxinA (Botox Cosmetic) and abobotulinumtoxinA (Dysport) have been approved by the US Food and Drug Administration?

A) Cervical dystonia
B) Forehead lines
C) Glabellar lines
D) Vertical lip creases

A

The correct response is Option C.

The only FDA-approved cosmetic indication for the use of onabotulinumtoxinA and abobotulinumtoxinA is to temporarily improve the appearance of moderate to severe glabellar lines. Although these products are widely used to treat other cosmetic concerns, these are all considered off-label uses. To this point, there have been no definitive adverse event reports of distant spread of botulinum toxin products when used at appropriate doses for dermatologic indications. There have been reports of adverse events in doses used for dystonia, especially in children. RimabotulinumtoxinB is only approved for treating cervical dystonia. On November 22, 2011, the FDA released a report renaming botulinum toxin type A and botulinum toxin type B to ensure their safe use (see table). Some of the reasons were to emphasize the differences in dosing and indications, and that these products are not interchangeable. The FDA recently approved application of Botox Cosmetic for smile lines related to activity of the lateral orbicularis oculi.

How well did you know this?
1
Not at all
2
3
4
5
Perfectly
37
Q

A 67-year-old woman with Fitzpatrick Type I skin comes to the office because of deep rhytides and signs of photoaging. Which of the following chemical peels will penetrate to the reticular dermis?

A) Alpha-hydroxy acid
B) Beta-hydroxy acid
C) Jessner solution
D) Phenol-croton oil
E) 20% Trichloroacetic acid

A

The correct response is Option D.

Croton oil is mixed with phenol to create a deeper peel. These are part of the ingredients in the Baker-Gordon and Hetter solutions. Hetter’s studies demonstrated that it was the croton oil that controlled the depth of the peel. The deeper the chemical peel, the greater the risk of scarring and hyperpigmentation. Because of this increased risk of hyperpigmentation, deep chemical peels are best suited for patients that have Fitzpatrick Type I skin.

Chemical peels vary in their depth of penetration into the dermis. Superficial peels penetrate to the epidermis. Alpha-hydroxyl acids (glycolic and lactic acid) and beta-hydroxy acid peels (salicylic acid) are superficial peeling agents. Jessner solution (14 g resorcinol, 14 g salicylic acid, 14 mL of lactic acid, and 100 mL of 95% ethanol) is also a superficial peel that can be used in conjunction with a trichloroacetic acid peel to achieve a deeper and more uniform peel. Superficial peels affect the epidermis and dermal-epidermal interface. Twenty percent trichloroacetic acid is a medium-depth peel that penetrates into the papillary dermis.

How well did you know this?
1
Not at all
2
3
4
5
Perfectly
38
Q

A 51-year-old woman comes to the office because of unilateral swelling of the breast 1 year after subglandular silicone augmentation mammaplasty. The patient says she has not had any recent trauma, fever, or myalgia. Physical examination shows a periprosthetic fluid collection. No erythema or edema is noted. Which of the following is the most appropriate next step in management?

A) Needle aspiration, Gram staining, and culture
B) Oral administration of antibiotics and complete blood count
C) Reduction of activity and use of a support brassiere
D) Ultrasound-directed aspiration and cytology

A

The correct response is Option D.

The most important diagnosis that needs to be ruled out is anaplastic large T-cell lymphoma (ALCL). This is a rare (one per million) non-Hodgkin lymphoma that has been reported in women with and without breast implants. However, increasing case reports suggest an association with breast implants, although direct causation has not been established. In ALCL associated with breast implants, malignant cells infiltrate the periprosthetic capsule or the periprosthetic fluid collection. The criteria for diagnosis include malignant cytology, strong CD30 expression, and cytokeratin negativity. Therefore, ultrasound-directed aspiration and cytology is most appropriate.

Reduction of activity, wearing a support brassiere, and follow-up in 2 weeks would be appropriate for perioperative tissue edema but is not appropriate treatment for late seromas.

Needle aspiration with Gram staining and culture risks injury to the implant and would not give the cytology necessary to determine if ALCL were present.

Oral administration of antibiotics and complete blood count would not be warranted in this situation where infection is unlikely by history and physical examination.

Ultrasound-directed aspiration and drain placement alone would not give the cytology necessary to determine if ALCL were present.

How well did you know this?
1
Not at all
2
3
4
5
Perfectly
39
Q

A 40-year-old woman comes to the office because of fine rhytides and skin discoloration caused by photoaging. Topical application of 0.025% tretinoin is planned. Which of the following best describes the mechanism of action of topical retinoid therapy?

A) Decreased activation of skin appendages resulting in thinning of the stratum corneum
B) Decreased corneocyte cohesion resulting in increased desquamation
C) Increased free radical scavenging activity and synthesis of Types I and III collagen
D) Inhibition of AP-1 transcription factor binding to DNA resulting in diminished protease activity
E) Inhibition of tyrosinase resulting in diminished conversion of dopamine to melanin

A

The correct response is Option D.

Tretinoin (all-trans-retinoic acid; Retin-A) is one of the best long-term topical therapies available for chronically photoaged skin. The mechanism of action of retinoids is regulated through specific nuclear receptors. Ultraviolet (UV) radiation activates a series of phosphokinases that stimulate c-Fos and c-Jun proto-oncogenes and thereby activate AP-1 transcription factor. AP-1 causes activation of metalloproteases such as collagenase, gelatinase, and stromelysin, which then break down collagen. Tretinoin results in a 70% inhibition of AP-1 transcription factor binding to DNA and a significant reduction in protease activity.

Long-term use of tretinoin is associated with improved skin texture, decreased sallowness, a reduction in fine rhytides and actinic keratosis, fading of pigmented macules, and an overall improvement in skin appearance. Histologic effects of tretinoin include increased epidermal and granular layer thickness; elimination of dysplasia, atypia, and microscopic actinic keratoses; uniform dispersion of melanin granules; increased collagen and glycosaminoglycan deposition in the papillary dermis; and diminished dermal elastosis, angiogenesis, and compaction/thinning of the stratum corneum.

Tretinoin side effects include erythema, photosensitivity, and desquamation. Patients are initially started on a low dose with nightly application until tolerance is achieved. Because tretinoin is a photosensitizer, sunscreen use is absolutely imperative. Topical retinoids should be used for a minimum of 3 to 4 months, with the greatest improvement after 1 year of use. Patients who use alpha-hydroxy acids concomitantly with topical retinoids will see a synergistic effect, and this combination is tolerated well in most patients.

Isotretinoin (13-cis retinoic acid; Accutane) impairs sebaceous gland activity, impairs epithelialization, and thins the stratum corneum. Alpha-hydroxy acids cause desquamation as a result of diminished corneocyte cohesion just above the granular cell layer. Dermal effects of these acids include increased collagen and glycosaminoglycan production. UV radiation-stimulated oxygen free radicals are the primary mediators of UV skin damage. Vitamin C has been shown to be the primary water-soluble nonenzymatic antioxidant that helps protect skin cells from UV radiation. Other functions of Vitamin C include increased Types I and III collagen production, decreased pigment synthesis, improved epidermal barrier function, and regeneration of oxidized Vitamin E. Hydroquinone impairs the conversion of dopamine to melanin. The result is decreased formation of melanin and increased degradation of melanosomes.

How well did you know this?
1
Not at all
2
3
4
5
Perfectly
40
Q

A 65-year-old woman comes to the office because of dysphagia and voice changes 3 days after undergoing injection of 50 units botulinum toxin type A because of platysmal banding. This patient’s condition is most likely caused by injection of botulinum toxin type A to which of the following anatomical locations?

A) Central fat pad below the thyroid cartilage
B) Cervicomental junction
C) Inferior border of the mandible at the angle
D) Pars facialis below the mandibular margin

A

The correct response is Option B.

In patients wishing to avoid platysmaplasty or surgical neck lift, platysmal bands can be softened with neuromodulators. The platysma is a very superficial muscle, and injecting neuromodulators too deeply may affect the strap muscles, causing dysphagia, or the cricothyroid muscle, causing voice changes. While injection into the pars facialis just below the mandibular margin is safe, the cervicomental junction is considered a danger zone because of the potential effect on deeper muscles involved in swallowing. Injection inferior to the thyroid cartilage and centrally would not effectively treat the banding but a superficial injection into the fat is unlikely to cause any muscular disturbance. Injection along the inferior border of the mandible at the angle may affect facial nerve function but would not cause dysphagia or voice change. Injection at the medial margin of the sternocleidomastoid is unlikely in the treatment of platysmal banding.

In a patient who presents with dysphagia or vocal changes after treatment of platysmal banding with neuromodulator, a reversible orally active anticholinesterase agent like pyridostigmine may be useful to counteract some of the effects until the agent wears off. Care must be taken to monitor for adverse side effects of anticholinesterase treatment, such as nausea, vomiting, diarrhea, and increased salivation.

41
Q

A 45-year-old woman comes to the office for consultation regarding smoothing of furrows between the eyebrows. Which of the following findings is a contraindication to abobotulinum toxin type A (Dysport) treatment in this patient?

A) Diabetes mellitus
B) Ehlers-Danlos syndrome
C) Gluten intolerance
D) ILupus
E) Milk allergy

A

The correct response is Option E.

Since the U.S. Food and Drug Administration approval and introduction of botulinum toxin type A in the 1990s, injectable cosmetic treatments have exponentially outnumbered cosmetic surgical treatments provided by plastic surgeons, and the prevalence of treatment continues to increase. Dysport is a product developed by Medicis (Scottsdale, AZ), which also has botulinum toxin as an active ingredient to reduce glabellar furrows. Dysport is contraindicated for individuals who have an allergy to cow’s milk protein.

42
Q

A 42-year-old woman comes for follow-up examination 1 day after undergoing injection of 0.8 mL of hyaluronic acid into the glabella. She has had severe pain since the procedure. Physical examination shows marked swelling and violaceous discoloration of the skin overlying the glabella. Massage and warm compresses produce minimal improvement. Which of the following is the most appropriate next step in management?

A) Antibiotic therapy
B) Aspiration of injected material
C) Conservative operative debridement
D) Hyperbaric oxygen therapy
E) Injection of hyaluronidase

A

The correct response is Option E.

Although the use of injectable soft-tissue fillers is relatively simple, it is not completely without risk. Patients should be consulted with respect to the possibility of the potential complications, and surgeons must be familiar with their incidence and treatment.

The patient in the scenario described has atypically severe pain, swelling, and discoloration 1 day after the procedure. While most patients will report some mild discomfort and bruising, these symptoms are usually not severe, and have usually begun to resolve within the first few days of the procedure. Severe persistent pain prompts further evaluation and potentially early intervention. The presence of persistent pain, swelling, and violaceous discoloration can be an indication of vascular compromise, which is most often seen after injection of the glabellar region. Direct arterial injection will result in immediate blanching in the distribution of the artery. Venous compression, on the other hand, results when too large a volume of filler is injected into a small area, resulting in venous congestion of the overlying skin.

Treatment of vascular compromise should be prompt and aggressive. If blanching is noticed on injection and an arterial occlusion is suspected, injection should be stopped and aspiration attempted. Patients with venous compromise usually have symptoms 1 or 2 days after the procedure, and aspiration at this point is rarely feasible. Massage and warm compresses should be initiated to disperse the contained material and vasodilate the compromised dermal vasculature. Nitroglycerin paste may also be helpful. Injection of hyaluronidase will further debulk the filler and is an important tool in the treatment of both early and late filler complications. If skin necrosis seems imminent, more aggressive measures, such as hyperbaric oxygen therapy, may be helpful. Once present, necrosis is treated with topical antibiotics and, potentially, surgical debridement.

Prevention of this complication is more likely to occur if the surgeon uses a smaller needle and the smallest possible volume of filler to correct the problem. Superficial injection should also be avoided. Patients should be informed of the potential complications of this procedure, and surgeons should be swift in the treatment of these problems when they arise.

43
Q

A hospital would like to run a marketing campaign for the use of botulinum toxin type A in the treatment of facial rhytides on behalf of the plastic surgery department. Which of the following areas of the face is most appropriate to address in this campaign?

A) Chin
B) “Crow’s feet”
C) Forehead
D) Glabella
E) Jowls

A

The correct response is Option D.

Botulinum toxin type A is Food and Drug Administration-approved only for use in the glabellar area. While off-label uses of FDA-approved drugs, such as the use of botulinum toxin type A in the treatment of forehead rhytides and brow ptosis, is legal, only FDA-approved uses may be advertised.

44
Q

Which of the following is the beneficial effect of pretreatment with tretinoin prior to facial chemical peel and laser resurfacing?

A) Decreased epidermal proliferation
B) Decreased fibroblast deposition of glycosaminoglycans
C) Increased collagen IV deposition
D) Increased epidermal melanin
E) Increased transit rate of keratinocytes through the epidermis

A

The correct response is Option E.

Tretinoin has long been established as a topical treatment that can improve photoaged skin by decreasing pigmentation and fine and coarse wrinkles. This is accomplished via activation of retinoic acid receptors and other not yet well-characterized molecular mechanisms. Epidermal hyperproliferation is induced, which results in a normalization of epidermal disarray and thickening of the epidermis. Additionally, fibroblast deposition of collagen and glycosaminoglycans is stimulated, increasing skin turgor and elasticity. The breakdown of collagen is reduced via the reduction of collagenase and promotion of collagenase inhibitors. Epidermal melanin is reduced because of the stable rate of melanin transfer from melanocytes to keratinocytes, whereas the transit rate of keratinocytes through the epidermis is increased.

45
Q

A 60-year-old woman comes to the office because she is unhappy with the thin appearance of her lips and wrinkles around her mouth (shown). Which of the following is the most appropriate management?

A) Implantation of acellular human dermis
B) Injection of botulinum toxin type A
C) Injection of hyaluronic acid
D) Injection of poly-L-lactic acid
E) Injection of silicone

A

The correct response is Option C.

Hyaluronic acid injectable products, such as Juvéderm, Restylane, or Perlane, are the best treatment available to improve fullness of the nasolabial fold and lip area. The products smooth wrinkles and depressions by filling the subcutaneous tissue. Volume deficiency is the essential problem with deep facial folds, particularly below the level of the eyes.

Implantation of acellular human dermis (AlloDerm) requires the risk of a surgical procedure without the promise of long-term results in lip augmentation. AlloDerm injectable Cymetra product is no longer available commercially and was plagued by its difficulty during injection due to high density.

Botulinum toxin type A (BOTOX Cosmetic) paralyzes muscles that animate the face and pull on the skin, resulting in deepening of the creases of the face. BOTOX Cosmetic does not fill deficient volume. In paralyzing the levator of the lip that deepens the nasolabial fold, BOTOX Cosmetic lowers the lip, enhancing the length of the upper lip and resulting in a more atrophic appearing lip. Poly-L-lactic acid (Sculptra) is a filler, but it is also known to result in nodule formation in dynamic muscles of the face, particularly around the lip and eye.

Silicone is not FDA-approved for injection in the United States because of its known risks of nodule formation and overlying skin inflammation and necrosis, which are progressive.

46
Q

A plastic surgeon is asked by a local charity to donate his services to an annual fundraiser. Which of the following medical services is currently permitted by the code of ethics of the American Society of Plastic Surgeons (ASPS)?

A) Augmentation mammaplasty
B) Liposuction
C) Injection of botulinum toxin type A
D) Rhytidectomy
E) No services are permitted

A

The correct response is Option C.

Injection of botulinum toxin type A does not require an incision; therefore, it is not a medical procedure as defined by the American Society of Plastic Surgeons (ASPS). The ASPS defines a “procedure” (Section 2, Article I [K] of the code of ethics) as a medical service that requires an incision. Examples of medical services that require an incision include, but are not limited to, rhytidectomy, augmentation mammaplasty, blepharoplasty, and suction lipectomy. Examples of medical services that do not require an incision include, but are not limited to, injections (botulinum toxin type A, hyaluronic acid), microdermabrasion, and other skin surface treatments. The other options listed all require an incision and are therefore prohibited as donations.

47
Q

A 45-year-old woman comes to the office because she desires facial rejuvenation. Various fillers, as well as botulinum toxin type A (BOTOX Cosmetic), are discussed as possible treatment options. As part of the discussion of informed consent, the plastic surgeon is obligated to tell the patient which of the following about the use of BOTOX Cosmetic?

A) Use of BOTOX Cosmetic and fillers in combination for perioral rejuvenation is approved by the FDA
B) Use of BOTOX Cosmetic and fillers in combination for periorbital rejuvenation is approved by the FDA
C) Use of BOTOX Cosmetic and fillers simultaneously is not approved by the FDA
D) Use of BOTOX Cosmetic for amelioration of platysmal banding is approved by the FDA
E) Use of BOTOX Cosmetic outside of glabellar frown lines is not approved by the FDA

A

The correct response is Option E.

Off-label use of drugs or medical devices is the common practice of using a device or drug in a manner not currently approved by the FDA. A widespread, common example is the use of BOTOX Cosmetic for chemodenervation of muscles other than those involved with glabellar rhytides. Strictly speaking, use of botulinum toxin type A to treat anything other than glabellar furrows is not FDA-approved, but it is considered legal. The combination of BOTOX Cosmetic and fillers used outside of approved locations is legal but considered ?off label,? and therefore is not FDA-approved. If BOTOX Cosmetic and filler are used in their approved locations simultaneously, there is no issue with FDA approval. The physician must understand that off-label protection is not an open approval for usage, but permits physicians to use medications in a manner different from that approved by the FDA as long as that use is not considered experimental. Generally, there should be evidence that the off-label use is safe by citing general acceptance in the medical community or by the presence of peer-reviewed publications supporting its use. The patient must be informed during the consent process that the use is ?off-label,? not FDA-approved, and acknowledge such by signing documentation during the informed consent. Manufacturers may not advertise off-label use to the public, but may distribute peer-reviewed publications to physicians as long as an appropriate disclosure is present identifying the off-label nature of that particular use and any potential conflicts of interest by the publication’s authors.

48
Q

Which of the following types of medication is most likely to potentiate the effect of botulinum toxin type A?

A) Angiotensin-converting enzyme (ACE) inhibitors
B) Beta-adrenergic blockers
C) Calcium-channel blockers
D) Diuretics
E) Nitroglycerin

A

The correct response is Option C.

BOTOX Cosmetic is used for improving forehead and glabellar lines as well as ?crow’s feet? and platysmal bands. The mechanism of action is via blocking neuromuscular transmission by binding to receptor sites on motor nerve terminals, entering the nerve terminals, and inhibiting the release of acetylcholine. The following medications are known to potentiate the effects of BOTOX Cosmetic: penicillamine, quinine, calcium-channel blockers, and aminoglycosides.

49
Q

A 56-year-old woman who has undergone injections of botulinum toxin type A (BOTOX Cosmetic) for orbital spasms for the past 16 years has developed resistance to the treatment. Botulinum toxin type B (Myobloc) is considered as an alternative. Compared with BOTOX Cosmetic, which of the following is most likely to result from treatment with Myobloc?

A ) Greater pain with injection

B ) Less effacement of the rhytides

C ) Longer duration of action

D ) Longer onset of action

E ) Lower radius of diffusion

A

The correct response is Option A.

Injection of botulinum toxin to improve rhytides is currently the most common cosmetic procedure performed in the United States. Although only BOTOX Cosmetic is currently approved by the FDA for cosmetic use in the United States, other toxins have been used abroad and will likely be available to practitioners in the United States in the near future. Myobloc has not yet received cosmetic approval but is approved by the FDA for treatment of cervical dystonia and hemifacial spasm. Given the concerns surrounding the potential for antibody development after chronic and repeated injection of BOTOX Cosmetic, in addition to the fact that serotypes do not cross-neutralize, there has been heightened interest in developing a neuroblocker that is effective for patients who may become resistant to BOTOX Cosmetic.

Myobloc has been studied for treatment of various muscle disorders, including cervical dystonia. Its effectiveness in the treatment of canthal rhytides has also been evaluated and compared to BOTOX Cosmetic in a double-blind randomized study. All major studies have shown that Myobloc safely and effectively produces a neuromuscular blockade and reduction in rhytides similar to BOTOX Cosmetic. There are, however, key differences between the two. Myobloc has an acidic pH of 5.6. Its low pH is the reason that it is stable in solution; for this reason, it is preconstituted in liquid, as opposed to BOTOX Cosmetic, which is packaged as a powder. Myobloc remains stable in liquid, providing a prolonged shelf life without diminished potency. It is also the acidic pH that accounts for a higher degree of pain on injection, which is noticed by most patients. Other differences that have been noted for Myobloc as compared to BOTOX Cosmetic are a shorter duration of action, quicker onset of action, and greater radius of diffusion. The duration effect may be secondary to inadequate dosing in the studies that have been performed to date. Optimal dosing for Myobloc for cosmetic applications has yet to be established.

The increased potential for diffusion with Myobloc may be seen as an advantage in certain circumstances and a disadvantage in others. While diffusion after injection into small muscle groups such as the glabellar region may lead to undesirable side effects (eg, ptosis), diffusion after injection of the platysma, for example, may lead to a more uniform and complete blockade. This characteristic may also be advantageous when injecting the axilla for the treatment of hyperhidrosis.

50
Q

A 35-year-old woman comes to the office for consultation regarding correction of her pronounced nasolabial folds. Soft-tissue fillers are considered. Which of the following injectables is most likely to result in a hypersensitivity reaction?

A ) Bovine collagen (Zyplast)

B ) Calcium hydroxylapatite (Radiesse)

C ) Human collagen (CosmoPlast)

D ) Hyaluronic acid (Juvéderm)

E ) Porcine collagen (Evolence)

A

The correct response is Option A.

Hypersensitivity reactions with soft-tissue fillers are relatively uncommon. Local reactions can occur with any of the fillers, but most reactions likely stem from local irritation associated with the technique of injection or quantity injected versus a true allergic reaction. In the case of Zyplast, approximately 3% of patients will exhibit a hypersensitivity reaction caused by a preexisting allergy to bovine collagen. Thus, for the bovine collagen products (Zyderm I and II; Zyplast), a skin test should be performed prior to use.

The other fillers listed have much lower incidences of reactions. Thus, they do not require skin tests prior to injection. CosmoPlast is derived from a single human fibroblast cell culture and is treated with glutaraldehyde to decrease its immunogenicity. Evolence is a porcine-derived collagen. It is much less immunogenic than bovine collagen. During its preparation, it is treated to remove allergenic telopeptides. Juvéderm is a hyaluronic acid-based product. Because it is not derived from animals, it is also associated with a lower risk of allergic reactions than the bovine product.

Juvéderm (a hyaluronic acid-based product) and Radiesse (calcium hydroxylapatite) are not derived from animals and are thus associated with a lower risk of allergic reactions than the bovine product.

51
Q

Which of the following acids is a component of Jessner solution?

A ) Ascorbic acid

B ) Glycolic acid

C ) Kojic acid

D ) Malic acid

E ) Salicylic acid

A

The correct response is Option E.

Jessner solution is a preparation used for light peels alone or in preparation for a trichloroacetic acid (TCA) peel. It contains resorcinol USP, salicylic acid USP, and lactic acid USP, 14% each in ethanol USP. It is a clear, pale-to-medium, yellow-pink solution with an odor of alcohol. Fresh resorcinol should be used because it turns dark with exposure to light and air. Salicylic acid is light sensitive. Trace amounts of ferric iron cause salicylic acid to turn red. Jessner peel solution should be kept in an air-tight amber bottle. Lactic acid absorbs water in moist air, and the use of old lactic acid can result in a subpotent preparation.

Kojic acid, which is not part of Jessner solution, is used as a skin lightener that works by inhibiting tyrosinase. Kojic acid is used in the treatment of melasma; side effects include slight skin irritation and possible development of sun sensitivity. Ascorbic acid, glycolic acid, and malic acid are not components of Jessner solution.

52
Q

A 45-year-old man comes to the office for consultation regarding deep nasolabial folds. He is interested in a product approved for use by the FDA, but it is difficult to obtain in the United States because the manufacturing company went out of business. The patient is able to obtain the product from Canada and brings a 1.4-mL syringe of the product to the office, requesting injection. Which of the following is the most appropriate action by the plastic surgeon?

A ) Do not inject the product because the company went out of business, revoking the product’s FDA-approved status

B ) Do not inject the product because it would be a violation of the Federal Food, Drug, and Cosmetic Act

C ) Do not inject the product because the patient obtained it from outside the United States

D ) Inject the product because it is approved by the FDA

E ) Inject the product because it was legally obtained

A

The correct response is Option C.

The act of importing drugs already manufactured or approved in the United States from another country is called reimportation. Currently, only manufacturers are permitted to reimport their own product. It would not be a violation of the Federal Food, Drug, and Cosmetic Act because the physician €™s conduct did not relate to the interstate delivery of a non-approved drug or device in the practice of medicine. Although the product is FDA-approved, it is illegal to inject it because of the way it was obtained.

53
Q

A 45-year-old woman with brow ptosis and periocular rhytides undergoes treatment with botulinum toxin type A (BOTOX Cosmetic). Injection of BOTOX Cosmetic into which of the following muscles is most likely to result in lateral brow elevation?

A ) Corrugator supercilii

B ) Depressor supercilii

C ) Frontalis

D ) Orbicularis oculi

E ) Procerus

A

The correct response is Option D.

Injection of BOTOX Cosmetic into the orbicularis oculi is most likely to result in lateral brow elevation. Facial rhytides of the periocular region are common aesthetic concerns resulting from hyperactivity of the facial musculature. Clinical studies have demonstrated the effectiveness of botulinum toxin in the treatment of periocular rhytides resulting from activity of the orbicularis oculi. Botulinum injections within the periorbital region can produce elevation of the brow.

The frontalis muscle is the primary elevator of the eyebrow. Muscles that antagonize elevation of the brow and therefore produce depression of the brow include the orbicularis oculi, corrugator supercilii, depressor supercilii and procerus. Laterally, the orbicularis oculi muscle acts a lateral eyebrow depressor. Botulinum toxin injections in the orbicularis oculi weaken the muscle and result in unopposed frontalis elevation of the lateral brow. Depression of the medial brow is produced by action of the corrugator supercilii, depressor supercilii, and the procerus. Weakening the action of these muscles will augment the primary function of the frontalis muscle and produce elevation of the medial brow.

54
Q

A 65-year-old woman comes to the office because she is dissatisfied with the aging appearance of her face. Physical examination shows a loss of facial volume and a moderate amount of skin laxity. Autologous fat grafting is planned. Which of the following techniques is most appropriate to maximize the “take” of the fat graft?

A ) Ensuring contact with other grafted fat with each successive pass

B ) Injecting a small amount of fat per pass

C ) Optimizing placement along the cortical facial bone

D ) Rinsing the graft with Ringer’s lactate

E ) Using standard suction lipectomy to harvest the graft

A

The correct response is Option B.

One of the main concerns after fat grafting is the potentially high rate of absorption in the grafted site over time, which may reach up to 70% of the filled volume. To maximize surface area contact between graft and vascularized bed tissues, the placement of extremely small amounts of fat with each pass is considered to be one of the keys to a successful fat graft €œtake. € Diffuse infiltration with multiple passes and attempts to separate the newly grafted fat parcels from each other are also considered important. Avoiding contact with other grafted fat with each pass means that more of the fat will be in contact with the vascularized bed tissue; this result stabilizes placed fat to deter migration and enhance integration. Placing fat directly on cortical bone may not enhance graft €œtake € because of the relatively poorer vascularity of the recipient bed. One animal study of the use of hyperbaric oxygen therapy following fat grafting failed to show any associated increased graft €œtake €; this technique has not been studied adequately in humans. Rinsing the graft with Ringer €™s lactate will have no effect on the graft €œtake. € The use of standard suction lipectomy harvest techniques has been shown to reduce viability of adipocytes within harvested fat grafts, compared with syringe or direct excision harvest.

55
Q

A 48-year-old man comes to the office for consultation regarding correction of deepened nasolabial folds. He is interested in long-lasting results but is able to miss only a few days of work. Treatment with which of the following is most likely to result in the longest recovery time?

A ) Calcium hydroxylapatite (Radiesse)

B ) Collagen (Zyderm)

C ) Fat graft

D ) Hyaluronic acid (Restylane or Juvéderm)

E ) Poly-L-lactic acid (Sculptra)

A

The correct response is Option C.

Patients who cannot tolerate excess post-treatment downtime are not ideal candidates for fat transfer procedures. These treatments are placed deeper in the dermis with large-gauge needles and can result in more significant bruising and swelling. For patients who require rapid recovery, the thinner hyaluronic acid products or even collagen-based products may be better choices. Radiesse is a long-lasting filler and has a rapid recovery time as well. Poly-L-lactic acid (Sculptra) is FDA-approved for the management of HIV facial lipoatrophy. It serves as a volume enhancer and is used for indications similar to those for autologous fat transfer.

56
Q

A 42-year-old man who is HIV positive comes to the office for management of facial lipoatrophy. A photograph is shown. Injection of which of the following is the most appropriate treatment?

A ) Bovine collagen
B ) Human collagen
C ) Hyaluronic acid
D ) Liquid silicone
E ) Poly-L-lactic acid

A

The correct response is Option E.

Lipoatrophy from HIV-associated lipodystrophy can be treated with numerous soft-tissue fillers, but only poly-L-lactic acid (Sculptra) is currently approved by the United States Food and Drug Administration (FDA). Sculptra typically is injected subcutaneously in the cheek, submuscularly in the orbital region, and subperiosteally as a €œdepot € in the temples (see the photograph below). Patients require multiple injections every four to six weeks for several months, but the results may last for over two years.

57
Q

A 45-year-old woman with Fitzpatrick type III skin comes to the office for a chemical peel to improve the appearance of sun-damaged facial skin. Initial treatment with tretinoin was unsuccessful. Examination shows fine rhytides over the cheek and perioral region. She has a history of perioral herpetic lesions. Which of the following is an advantage of using a trichloroacetic acid peel rather than a phenol peel in this patient?

A ) Consistent penetration into the upper reticular dermis

B ) Deeper penetration into the dermis

C ) Increased effectiveness in reducing coarse facial rhytides

D ) Less risk for outbreak of herpetic lesions

E ) Less risk for pigmentary changes

A

The correct response is Option E.

A phenol peel causes more hypopigmentation than a trichloroacetic acid (TCA) peel. This is more likely to occur in patients with darker skin or in patients who have been exposed to the sun after undergoing a phenol peel. First-line treatment for rhytides is tretinoin, hydroquinone, or glycolic acids. Treatment for refractory cases is a medium-depth TCA peel.

Phenol peels provide a controlled, predictable chemical injury with consistent penetration to the upper reticular dermis. A deep TCA peel has only one half the degree of penetration into the dermis when compared to a phenol peel. Because of this lack of penetration, even deep TCA peels do not have as profound an effect on coarse facial rhytides. The depth of the phenol peel is associated with a significant bleaching action and hypopigmentation.

Patients who have a history of perioral herpetic lesions require pretreatment with acyclovir before any chemical peel procedure. In the absence of pretreatment, a patient with a history of perioral herpetic lesions has a 50% chance of developing an outbreak. Even when pretreated with acyclovir, a patient with a history of herpetic lesions has an 8% chance of developing an outbreak; however, a patient with no history of perioral herpetic lesions who does not receive pretreatment with acyclovir has a 6.6% chance of developing an outbreak. The current recommendation is for all patients to receive pretreatment. Fortunately, the presence of a herpetic outbreak after a chemical peel, if treated promptly, will rarely leave residual scarring.

58
Q

A 63-year-old woman comes to the office for consultation regarding improvement of facial rhytides. A chemical peel is planned. Cardiac monitoring is required if which of the following chemical solutions is applied during the procedure?

A ) 50% Glycolic acid

B ) Jessner solution

C ) 50% Lactic acid

D ) 30% Phenol-0.8% croton oil

E ) 35% Trichloroacetic acid

A

The correct response is Option D.

Phenol is a protoplasmic toxin that disrupts cell walls and denatures proteins. Dermal absorption is rapid, even through intact skin. High concentrations of phenol disrupt the dermal barrier and penetrate skin effectively. Its local anesthetic action produces a relatively painless coagulum. Although a small amount of phenol undergoes conjugation with glucuronic and sulfuric acids within one to two days of exposure, most is excreted unchanged in the urine. The half-life of phenol is estimated at 3.5 hours. Although it is infrequent, cardiac dysrhythmia is considered one of the potential complications of phenol peel. These dysrhythmias can include sinus tachycardia, premature ventricular contractions, atrial fibrillation, and ventricular fibrillation. Most of these dysrhythmias occur during the application of the phenol peel; therefore, appropriate cardiac and respiratory monitoring of the patient is essential.

Cardiac arrhythmias are not a reported complication when using trichloroacetic acid, glycolic acid, or Jessner solution.

59
Q

A 35-year-old woman is scheduled to undergo repair of a tear trough deformity using injections of hyaluronic acid. To minimize the risk for potential complications, which of the following is the most appropriate recommended level of infiltration of this filler?

A ) Superficial dermis

B ) Deep dermis

C ) Subcutaneous

D ) Intramuscular

E ) Periosteum

A

The correct response is Option E.

Many options are available to manage a tear trough deformity. The advantage of using injections of filler material is that this method avoids the downtime of surgical options such as prostheses and pedicled fat. Injections of fat are an option but can have unpredictable outcomes. Injections of hyaluronic acids have the advantage of little downtime, low risk, and, for the most part, self-limiting complications. When injecting hyaluronic acids for tear trough correction, the filler should be injected at the level of the periosteum. Injection at this site limits the risk of palpability and surface irregularity associated with injections at more superficial levels, such as the superficial and deep dermis, subdermal, or suborbicularis oculi fat. It also potentially improves the longevity of the filler because it is placed at a level where the tissue is immobile, thus negating the effect of animation to break down the product.

60
Q

A 67-year-old woman comes to the office because of sagging of her right upper eyelid two days after receiving injections of 50 U of botulinum toxin type A (BOTOX Cosmetic) to the right forehead and bilateral lateral orbital rims for treatment of rhytides and bilateral crow’s feet. Examination confirms ptosis of the right eyelid and resolution of the rhytides. Inhibition of which of the following muscles most likely caused this finding in this patient?

A ) Corrugator supercilii

B ) Frontalis

C ) Levator palpebrae

D ) Müller

E ) Orbicularis oculi

A

The correct response is Option B.

The patient discussed most likely developed induced ptosis secondary to inhibition of the frontalis muscle on the right. Hyperactivity of the frontalis muscle and secondary deep furrow formation likely developed as a compensatory means of correcting an undiagnosed right eyelid ptosis. By blocking frontalis function, the patient was no longer able to assist in enhancing the vertical palpebral aperture, and secondary upper eyelid ptosis occurred. This is an off-label use of BOTOX Cosmetic.

Treatment of BOTOX Cosmetic-induced eyelid ptosis includes administration of alpha-adrenergic eyedrops such as Iopidine, which causes stimulation and contraction of Müller muscle, a sympathetically innervated smooth muscle that elevates the upper eyelid by approximately 2 mm. The levator palpebrae, the major elevator of the upper eyelid, is a striated muscle innervated by the oculomotor nerve (CN III), and its function is unaffected by adrenergic stimulation.

An understanding of the anatomic and functional relationship of the muscles responsible for the dynamic lines of facial expression is mandatory to avoid complications in BOTOX Cosmetic injection. The main depressors of the eyebrow are the corrugator supercilii and the procerus muscle medially, and the superolateral orbicularis oculi laterally. The main antagonist of the eyebrow depressors is the frontalis muscle. Inhibition of the brow depressors would be unlikely to result in eyebrow or eyelid ptosis.

The most common mechanism for most upper eyelid ptosis that occurs after treatment with BOTOX Cosmetic relates to the diffusion of drug through the orbital septum to the levator muscle and less commonly the Müller muscle, which usually occurs with primary treatment to the glabellar region, and therefore is unlikely in the scenario described.

61
Q

Which of the following anatomic structures most effectively neutralizes trichloroacetic acid (TCA) in patients undergoing TCA peels?

A ) Epidermis

B ) Dermis

C ) Subcutaneous tissue

D ) Kidney

E ) Liver

A

The correct response is Option B.

In patients undergoing chemical peeling, the TCA is neutralized within the superficial dermal plexus, particularly by the protein keratin. Chemical peeling by TCA is associated with absence of systemic toxicity and adverse effects on the cardiovascular system. Therefore, these patients do not require monitoring using electrocardiogram during the procedure.

Phenol peels are absorbed in the systemic circulation to some extent. As a result, blood phenol level should be monitored. Cardiac irregularities are associated with phenol peel. Therefore, it is recommended that cardiac monitoring be performed during the procedure. Phenol is eliminated and excreted by the kidneys.

62
Q

A 45 year old woman comes to the office because she developed several painful vesicles on the lips since undergoing a trichloroacetic acid peel for rejuvenation of the perioral area five days ago. Medical history includes a similar outbreak two years ago. Which of the following is the most appropriate management?

(A) Oral prednisone

(B) Oral valacyclovir

(C) Topical bacitracin

(D) Topical hydrocortisone

(E) Reassurance and observation

A

The correct response is Option B.

Patients with a history of facial herpetic infections are at high risk for recurrence following chemical peel or laser resurfacing for facial rejuvenation and should receive antiviral prophylaxis with acyclovir or valacyclovir. The patient described likely has such an infection and should receive aggressive treatment to shorten the course of the infection.

Corticosteroids and bacitracin will not provide definitive therapy but may lessen symptoms.

63
Q

A 48-year €‘old woman comes to the office because she has painful lesions on the upper lip and medial aspect of the cheek three days after undergoing dermabrasion for treatment of fine rhytides and acne scarring. Physical examination shows red, blotchy, and weeping areas over the treatment sites. Which of the following is the most likely causative organism?

(A) Candida albicans

(B) Herpes simplex type 1

(C) Staphylococcus aureus

(D) Streptococcus pyogenes

(E) Varicella zoster

A

The correct response is Option B.

Outbreaks of herpes simplex virus (HSV) type 1 may occur after any dermal resurfacing procedure, including dermabrasion, chemical peels, and laser treatments. Reactivation of the virus is reported to occur in up to 50% of patients known to have a history of HSV and undergo resurfacing without prophylaxis treatment. Outbreaks occurred in 6.6% of patients without a known history of HSV; therefore, it is important that all patients undergoing a resurfacing procedure receive prophylaxis with antiherpetic medication (acyclovir, valacyclovir, or famciclovir) starting one to two days before the procedure and continuing until reepithelialization is complete.

Bacterial or candidal infections also can occur, although they are not as common as HSV infection. Perioperative antibiotics, careful antibacterial facial prepping, and frequent post €‘procedure handwashing are also important aspects of care.

Varicella zoster is not commonly associated with dermabrasion.

64
Q

A 53-year-old man who is HIV positive comes to the office for management of facial lipoatrophy. A photograph of the face is shown. Injection of which of the following fillers is the most appropriate treatment?

(A) Calcium hydroxyapatite

(B) Human-based collagen

(C) Hyaluronic acid

(D) Liquid silicone

(E) Poly €‘L-lactic acid

A

The correct response is Option E.

Facial lipoatrophy can affect as many as 50% of individuals infected with HIV. With more than 42 million people living with HIV worldwide, the demand for an effective, safe treatment of this stigmatizing deformity is increasing.

Many treatments for HIV €‘associated facial lipoatrophy have been explored. The ideal filler would be completely nonallergenic, produce an excellent aesthetic result, and have a long duration of effect but not be so permanent that errors in treatment could not be corrected.

Poly €‘L €‘lactic acid (Sculptra) is currently the only soft-tissue filler specifically approved by the United States Food and Drug Administration (FDA) for soft €‘tissue augmentation in HIV €‘associated lipoatrophy. Several recent studies in the United States have found that the use of Sculptra results in a significant and prolonged improvement in facial atrophy associated with HIV. The effect can last up to two years in some patients. There have been no reported cases of serious adverse reactions, and the treatment is well tolerated.

Although calcium hydroxyapatite (Radiesse), human-based collagen (CosmoPlast), and hyaluronic acid (Restylane) all have been used effectively in the treatment of this problem, none are as long €‘lasting as Sculptra. Moreover, placement of Radiesse in a more superficial plane can result in palpable nodularities that may require surgical excision.

The use of FDA €‘approved forms of liquid injectable silicone are currently being tested for the treatment of HIV €‘associated facial atrophy. Early results are promising, but longer follow €‘up of treated patients is required to adequately assess the efficacy, durability, and long €‘term safety of liquid injectable silicone for treatment of this disease.

65
Q

A 32-year old woman comes to the office for consultation regarding wrinkling along the sides of her nose. She received botulinum toxin (Botox) injections for forehead lines, glabellar furrowing, and crow’s feet one month ago. Physical examination shows vertical rhytides on the nasal sidewalls. Contraction of which of the following muscles is the most likely cause of these findings?

(A) Corrugator

(B) Frontalis

(C) Nasalis

(D) Orbicularis oculi

(E) Procerus

A

The correct response is Option C.

Activity of the transverse portion of the nasalis muscle causes bunny lines.

The frontalis muscle and the corrugators are responsible for forehead lines and glabellar lines, respectively. The lateral portions of the orbicularis oculi are responsible for crow €™s feet. The procerus is responsible for horizontal nasal lines, but not wrinkles on the sides of the nose.

66
Q

A 42-year-old woman comes to the office for consultation regarding removal of transverse wrinkles over the bridge of the nose. Administration of botulinum toxin type A is planned. Targeting of which of the following muscles is most appropriate?

(A) Corrugator supercilii

(B) Frontalis

(C) Orbicularis oculi

(D) Procerus

(E) Zygomaticus minor

A

The correct response is Option D.

The procerus muscle produces transverse wrinkles over the bridge of the nose. The corrugator supercilii serves to draw the eyebrow inferiorly and medially, and as such produces the vertical glabellar frown lines. The main antagonist of all of the eyebrow depressors is the frontalis muscle. Contraction of the orbicularis oculi enhances the radially oriented folds at the lateral canthus. The zygomaticus major muscle draws the angle of the mouth superiorly, laterally, and posteriorly with actions of laughing, smiling, and chewing. The zygomaticus minor muscle functions as one of the lip elevators and, with the zygomaticus major, contributes to the nasolabial fold.

67
Q

A 60-year-old woman with Fitzpatrick type 1 skin and coarse panfacial rhytides comes to the office for rejuvenation. Which of the following is the most effective treatment?

(A) Dermabrasion

(B) Glycolic acid peel

(C) Jessner solution peel

(D) Phenol peel

(E) Trichloroacetic acid (TCA) peel

A

The correct response is Option D.

Trichloroacetic acid (TCA) chemical peels are excellent for treating rhytides of the infraorbital skin and the full face. TCA concentration can be varied, from 10% to 25% for a light peel, 30% to 35% for an intermediate peel, and 50% to 60% for a deep peel. TCA is more effective than other treatments for neck, chest, and hands because the concentration can be lowered to avoid hypertrophic scarring, which may occur in these areas. Although TCA peels result in less facial bleaching than phenol peels, TCA cannot provide as profound an effect as phenol. Therefore, phenol is better for coarse wrinkles, severe sun damage, and significant pigment problems.

Although TCA peels can provide varying levels of treatment, the response to phenol peels is “all €‘or €‘none.” Histologic studies show that phenol provides double the penetration and double the amount of neocollagen formation compared with a deep TCA peel. Phenol results are long lasting. Problems associated with phenol include the bleaching effect (making it less useful for darker complected individuals), prolonged recovery period, and potential for cardiac toxicity.

Performed with a motor €‘driven diamond fraise or steel brush, dermabrasion is a technique of skin abrasion that can sand down scars or sculpt the surrounding tissue to a level where the defect becomes less noticeable. Dermabrasion removes the entire epidermis and the upper and mid-dermis, including the upper part of skin adnexa. Depth of treatment is controllable and may be varied to meet the needs of the patient; deeper penetration can improve deeper scars. Dermabrasion is thus better than peels for perioral rhytides and acne scarring. Re-epithelization and repigmentation take place from the residual portions of the hair follicles, sebaceous glands, and sweat ducts. Dermabrasion has less of a bleaching effect than phenol, so it is better for patients with darker complexions who need improvement in rhytides of the upper and lower lip and chin. Dermabrasion cannot be safely performed around the eye and is difficult to perform for full face treatment.

Glycolic acid peels and Jessner solution peels only provide treatment for superficial wrinkles, acne scars, and uneven pigmentation.

68
Q

Which of the following injectable fillers has the highest concentration of hyaluronic acid per volume of injectate?

(A) CosmoDerm

(B) CosmoPlast

(C) Radiesse

(D) Restylane

(E) Sculptra

A

The correct response is Option D.

Restylane is a U.S. Food & Drug Administration (FDA) €‘approved non €‘animal €‘stabilized hyaluronic acid derivative used for soft-tissue augmentation. Unlike Hylaform gel, it is derived from streptococcal bacterial fermentation and does not require an animal source. At 20 mg/ml, Restylane has a higher concentration of hyaluronic acid than Hylaform gel. It is used to treat rhytides and scars and in lip augmentation. Restylane correction was noted to be 82% at three months and 33% at one year in a study involving 285 wrinkles treated in 113 patients.

Hylaform gel, previously known as Hylan B gel, is a form of cross €‘linked hyaluronic acid (5.5 mg/ml) derived from the rooster combs of domestic fowl. It is reported to be less immunogenic and longer lasting than bovine collagen. One study showed 86% histologic persistence of Hylaform gel compared with 25% of bovine collagen in guinea pigs at 26 weeks. Besides being longer lasting than collagen, proponents say it has less risk of clumping and goes in more smoothly. Overcorrection is not needed with this product.

CosmoDerm and CosmoPlast are FDA €‘approved fillers consisting of natural human collagen grown under controlled laboratory conditions. Pretreatment skin tests are not necessary, and the materials are screened for purity. Both products contain 35 mg/ml of human €‘derived collagen in phosphate €‘buffered physiologic saline containing 0.3% lidocaine. CosmoDerm is not cross €‘linked and is used to treat superficial lines and wrinkles, whereas CosmoPlast is cross €‘linked with glutaraldehyde and can be used for deeper wrinkles.

Radiesse is composed of microspheres of calcium hydroxyl apatite suspended in an aqueous gel carrier. These biodegradable microspheres serve as a lattice upon which the body forms a scaffold for tissue infiltration. The spheres degrade slowly over years for a longer €‘lasting, semipermanent effect.

Sculptra is a poly €‘l €‘lactic acid that is FDA approved for management of HIV facial lipoatrophy. It serves as a volume enhancer and is used for indications similar to those for autologous fat transfer. Results are not immediate; treatment is performed as a series of three to five treatments approximately one month apart.

69
Q

A 41-year-old woman has been undergoing a series of intense pulsed-light therapy to treat fine wrinkles and areas of sun-induced hyperpigmentation on the face. She is pleased with the lightening of the brown spots but would like to see more improvement in the fine lines and skin texture. The patient €™s work schedule prohibits time off for recovery. Which of the following treatments is most appropriate for this patient?
(A) 4% Hydroquinone
(B) 30% Glycolic acid
(C) 30% Trichloroacetic acid
(D) 33% Phenol peel
(E) Jessner’s solution

A

The correct response is Option B.

A 30% glycolic peel is the best choice for this patient, who does not want a lengthy recovery period. Depth of injury will be limited to the stratum corneum and perhaps mild epidermal peeling. The patient must realize that her result will be more subtle than with a deeper peel. A series of treatments at monthly intervals is advisable for maintenance therapy. Glycolic acid is an alpha-hydroxy acid that promotes superficial desquamation and may stimulate cell turnover and collagen production. Solutions of 50% or 70% may be used for deeper peels. Depth of injury is also time-dependent, and the acid must be neutralized or washed off with water.

A trichloroacetic acid peel of 30% is stronger than a comparable strength of glycolic acid. This peel causes a medium-depth injury, extending to the papillary dermis, resulting in a period of erythema and epidermal healing that will be unacceptable to this patient. Trichloroacetic acid treatments are effective for skin tightening and mild hyperpigmentation. Strengths of this treatment typically range from 15% to 35%.

Phenol peels are also medium- to deeper-level peels that require a longer recovery time for healing. Hydroquinone is a melanin inhibitor that is used primarily for the management of hyperpigmentation. Jessner €™s solution (14% lactic acid, 14% resorcinol, and 14% salicylic acid) is often used in combination with trichloroacetic acid to create a medium-depth peel. Concerning adverse effects, studies have shown that skin treated with Jessner €™s solution showed a significantly increased degree of exfoliation when compared with glycolic acid.

70
Q

In a 65-year-old woman with Fitzpatrick type II skin, which of the following agents is the most effective single treatment of moderate facial rhytides?
(A) Ascorbic acid
(B) Glycolic acid
(C) Lactic acid
(D) Retinoic acid
(E) Trichloroacetic acid

A

The correct response is Option E.

Tretinoin and alpha-hydroxy acids, which include glycolic acids, are dependable components of any skin rejuvenation program. These products are well studied, and the results are predictable. Some improvement in fine wrinkles has been observed, although clear histologic improvement in photoaging remains controversial, with the primary clinical observations showing improvement in skin texture and €œclarity. € Alpha-hydroxy acids, such as glycolic acid, are commercially available for in-office application as peeling agents on a regular schedule. Antioxidants such as ascorbic acid (vitamin C) have shown some clinical promise in the prevention of photoaging. Trichloroacetic acid (TCA) produces superficial exfoliation in concentrations of 15% to 35% and is appropriate for the treatment of more defined wrinkles.

Hyaluronic acid is currently being studied as a useful adjunct in the treatment of depressed scars and facial irregularities. It is used as a filler substance for augmentation of lips and to correct lines and wrinkles as well. Available in Europe, South America, and Canada, it is not yet approved for use in the U.S.

Although TCA peels are generally regarded as safer than phenol peels, informed consent must include the potential complications of all chemical peels. Depth and consistency of peel with TCA is variable and depends on pretreatment with tretinoin/hydroquinone, strength of solution (10% to 25% for light peels, 30% to 35% for intermediate peels, and 50% to 60% for deep peels), and duration of application (sparse and pinkish white changes for superficial peel, dense white frosting for intermediate to deep peels). Patients with a history of perioral herpes simplex should be pretreated for 24 hours before and five days after chemical peel to decrease potential for herpetic reactivation and superinfection. TCA peels should be administered only by a physician.

71
Q

A 67-year-old woman comes to the office for consultation regarding prominent nasolabial folds. Injection of which of the following agents is the most appropriate management?
(A) Botulinum toxin
(B) Hyaluronic acid
(C) Hydroxyapatite
(D) Liquid silicone

A

The correct response is Option B.

Hyaluronic acid (Restylane, others) is an injectable product that acts as a temporary soft-tissue filler. It has been shown to be useful in improving the appearance of nasolabial folds, lip augmentation, and other signs of facial aging.

Botulinum toxin (Botox) is a neuromuscular blocking agent and not a soft-tissue filler. It is useful for temporary improvement of dynamic skin deformities associated with muscle contractions in the brow, forehead, and lateral orbital areas.

Hydroxyapatite is a permanent product that is used to augment bone structures such as the malar area or the mandible. Because it becomes a firm substance, its use in soft tissue such as the nasolabial folds is not recommended.

Medical-grade liquid silicone (Adatosil 5000, Silikon 1000) is approved by the U.S. Food and Drug Administration (FDA) for intraocular ophthalmic injections but not for use as a soft-tissue filler. Previous experience with silicone injections has resulted in long-term complications that are difficult to treat. Despite its unfavorable history, liquid silicone injections are becoming popular again. Proponents claim the new liquid silicone is more pure than that used in the past. They also recommend the microdroplet (0.01 to 0.04 ml) technique of injection, because larger injection doses produce foreign-body granuloma reaction. Given the potential complications, the difficulty of removing it from soft tissue, and the lack of FDA approval, the use of liquid silicone should wait until long-term studies are able to provide more information on the risks and benefits of its use.

72
Q

A 50-year-old woman has ptosis of the upper eyelids after undergoing injection of botulinum toxin to the glabellar region. This finding is most likely due to paresis of which of the following periorbital muscles?
(A) Corrugator
(B) Levator
(C) Mullers
(D) Orbicularis
(E) Procerus

A

The correct response is Option B.

Use of botulinum toxin for management of hyperkinetic frown lines and furrows is an effective primary, adjunctive, or prophylactic therapy for patients desiring facial rejuvenation. Adverse effects can be minimized with a thorough understanding of the facial soft-tissue anatomy, proper patient selection, and administration of the lowest effective doses with minimal volume of delivery.

The treatment of glabellar frown lines typically involves injection of toxin into the medial eyebrows (corrugators). Other targeted facial hyperkinetic lines include lateral canthal rhytides (crow €™s feet) and horizontal forehead furrows. Diffusion of the toxin into the levator muscles upon injection of toxin into the corrugator muscles causes blepharoptosis. This adverse effect typically lasts only a few weeks because the dose of migrated toxin to the affected muscle is reduced significantly from the site of initial injection.

Injection into the procerus at the central glabella or into the frontalis muscle for horizontal forehead furrows would not likely lead to involvement of the levator muscle of the eyelid. Similarly, injection of the lateral portion of the orbicularis oculi at the outer canthal rhytides would not lead to the findings described in this patient. Müller €™s muscle lies deep to the levator muscle and is less likely to be injured than the levator during eyelid procedures.

73
Q

In preparation for fat injection, which of the following is the most appropriate technique for processing the lipoaspirate to yield the highest volume of viable fat cells?
(A) Balanced centrifugation
(B) Exposure to air
(C) Filtration
(D) Gravity sedimentation
(E) Rinsing with isotonic saline

A

The correct response is Option A.

Balanced centrifugation is the appropriate technique for preparing lipoaspirate. The sediment of the harvested material separates reliably with brief centrifugation. Filtration, rinsing, or straining should not be performed because these methods lead to disruption of the fragile fat cells. Air exposure, even briefly, results in cytoplasmic lysis, and drying should be avoided. Gravity sedimentation is the optimal process but is lengthy. Duration of one to two hours ex vivo would increase the risk of drying and lipolysis.

74
Q

A 30-year-old woman comes to the office for consultation regarding deep Aice-pick@ scarring of the cheeks. Which of the following is the most effective management?
(A) Intense pulsed-light therapy
(B) Glycolic acid peel
(C) Microdermabrasion
(D) Collagen injection
(E) Direct excision

A

The correct response is Option E.

Ice-pick scarring is a full-thickness injury to the skin and does not respond well to superficial treatments. The best option is direct excision and closure of each ice-pick scar.

Intense pulsed-light (IPL) therapy may improve the collagen in the dermis but will not correct a full-thickness injury.

Glycolic acid peels and microdermabrasion have their main effect on the epidermis. These modalities may reduce fine rhytides but will not affect ice-pick scars.

Collagen injections have been used to correct scars associated with acne, but the results are temporary.

75
Q

In micrografting hair transplantation, which of the following best represents the structure of the transplanted unit?
(A) Isolated hair follicles
(B) Hair follicles with dermal elements
(C) Hair follicles with subcutaneous tissue
(D) Hair follicles with galea
(E) Hair follicles with pericranium

A

The correct response is Option B.

Hair in healthy scalp grows in one, two, three, or four hairs, each with their own associated neurovascular bundles, sebaceous glands, sweat glands, and piloerectile muscles surrounded by collagen. These €œphysioanatomic € units, when used as micrografts, have been shown to provide excellent results in hair transplantation.

The anatomy of an individual hair follicle includes the dermal papillae bulb, consisting of the dermal and epidermal coat. The dividing cells within the bulb form a cement column of keratinized dead cells held together with a cystine matrix to make the hair shaft.

The macroscopic hair transplantation technique of hair plugs with multiple hair follicles, intervening skin, subcutaneous tissue, epicranial and subepicranial tissue can successfully transplant hair but with an unnatural appearance.

76
Q

A healthy 55-year-old woman comes to the office for consultation regarding reduction of frown lines between her eyebrows. Injections of botulinum toxin are planned. The patient has never undergone this treatment. Which of the following is the most appropriate initial dose of botulinum toxin type A for injection into the glabellar region in this patient?
(A) 1 U
(B) 5 U
(C) 20 U
(D) 45 U
(E) 100 U

A

The correct response is Option C.

Clostridium botulinum toxin type A (Botox) is supplied in a vial containing 100 U of vacuum-dried neurotoxin complex. The recommended starting dose of botulinum toxin type A for improvement of glabellar wrinkles is 20 U. Prescribing information recommends five injection sites along the medial brow and glabellar region. The corrugator supercilii, procerus, and depressor supercilii are the muscles of the glabellar region that are targeted. The most common complication in treatment of the glabellar complex is ptosis of the upper eyelid.

77
Q

Which of the following is the most appropriate method for decreasing the depth of a phenol chemical peel?

(A) Applying antibiotic ointment
(B) Taping the skin
(C) Using croton oil
(D) Using liquid soap

A

The correct response is Option D.

The depth of a phenol peel is decreased by using liquid soap, which increases the surface tension and thus decreases the penetration and absorption of the phenol.

Applying antibiotic ointment and taping the skin increase the depth of a phenol peel by providing a vapor barrier.
Phenol peels are typically based on croton oil, a skin irritant that increases the speed and depth of epidermal destruction. Phenol peels were previously performed according to the Baker formula, which involved placement of the phenol in a solution of distilled water, croton oil, and liquid soap. This formula is no longer used because the solution did not effectively dilute the toxicity of the phenol.

78
Q

In patients undergoing dermabrasion, which of the following is the most likely finding at the level of the superficial reticular dermis, indicating the endpoint of treatment?

(A) Coarse, nonbleeding tissue
(B) Confluent bleeding with a coarse tissue background
(C) Smooth, nonbleeding tissue
(D) Sparse, punctate bleeding with a pink tissue background

A

The correct response is Option B.

Dermabrasion, which is a method of skin resurfacing involving controlled mechanical abrasion of the epidermis and a variable portion of the dermis, is especially useful for treatment of scars, facial rhytides, and the skin deformities associated with rhinophyma. This process results in mechanical depression of scars and elevated tissues and induction of collagen synthesis. Following dermabrasion, wound repair begins in the remnant dermal appendages.

The depth of dermabrasion is determined by clinical endpoints encountered during the treatment process. The epithelium is removed initially; the dermal-epidermal surface is smooth and does not bleed. Sparse, punctate bleeding indicates that the surgeon has reached the superficial papillary dermis. This sparse bleeding becomes greater as the surgeon planes deeper into the papillary dermis, and the background is notably coarser. The level of the superficial reticular dermis is characterized by brisk, confluent bleeding on a coarse tissue background. This indicates the endpoint of treatment because further dermabrasion can lead to permanent scarring.

Although re-epithelialization typically occurs within seven to 10 days following dermabrasion, erythema may persist for as long as six weeks after treatment.

79
Q

Which of the following best describes the mechanism of action of botulinum toxin (Botox)?

(A) Inhibition of acetylcholine release at the neuromuscular junction
(B) Inhibition of messenger RNA-mediated production of acetylcholine
(C) Potentiation of the acetylcholine effect at the neuromuscular junction
(D) Prevention of acetylcholine binding at the neuromuscular junction

A

The correct response is Option A.

Botulinum toxin, also known as Botox, is an exotoxin derived from Clostridium botulinum bacteria. Its neuromuscular mechanism of action involves inhibition of the release of acetylcholine. Botulinum toxin is packaged (typically in units of 100) in a sterile, vacuum-dried form, which must be stored at -5%C (23%F). It can also be reconstituted by diluting it with nonpreserved saline at a rate of 2.5 U/.1 mL.

Botulinum toxin is currently approved by the Food and Drug Administration (FDA) for treatment of glabellar rhytides. However, it is increasingly being used as an “off label” treatment for dispersion of rhytides of the forehead and the periorbital, perioral, and platysmal regions.

80
Q

Which of the following is the most likely result of treatment of the skin with topical tretinoin (retinoic acid)?

(A) Cellular atypia
(B) Consolidation of melanin granules
(C) Decreased collagen synthesis
(D) Thinning of the epidermis
(E) Thinning of the stratum corneum

A

The correct response is Option E.

Tretinoin, or retinoic acid, is used for antiaging therapy because it can accelerate the reversal of skin damage caused by sun exposure. Its mechanism of action is believed to involve stimulation or oppression of specific genes within nuclear receptors. Tretinoin has been shown to inhibit the binding of AP1 transcription factor to DNA by 70%, thereby decreasing the activation of collagenase, gelatinase, and stromelysin.

Histologic effects associated with tretinoin use include thinning of the stratum corneum, reversal of cellular atypia, and thickening of the epidermis. In the dermis, collagen synthesis is increased and melanin granules are dispersed more evenly.

81
Q

A patient is scheduled to undergo injection of bovine collagen (Zyderm) in the glabellar region. A test dose should be administered how many weeks prior to injection?

(A) 1
(B) 2
(C) 4
(D) 8
(E) 12

A

The correct response is Option C.

Zyderm and Zyplast are highly purified forms of bovine collagen typically injected to provide improvement of rhytides and depressed scars. Because approximately 3% of all treated patients will have an allergic reaction to injectable bovine collagen, skin testing must be performed prior to any treatment. Following intradermal injection of a single test dose, the patient should be assessed 48 hours after injection and again at four weeks after injection. Any adverse changes noted at the test site indicate an allergic reaction, and occasionally reactions appear as long as four weeks after injection. Recently, some physicians have begun to advocate two episodes of antigen exposure, potentially increasing the number of positive reactions and identifying additional patients who may subsequently develop allergic reactions.

82
Q

A 37-year-old woman has symptomatic blepharoptosis on the right one week after undergoing injection of 25 units of botulinum toxin (Botox) into the glabellar region for treatment of dynamic rhytides. Which of the following is the most appropriate management?

(A) Observation
(B) Use of a patch on the left eye for 48 hours
(C) Use of alpha-adrenergic agonist eyedrops
(D) Administration of a beta-adrenergic blocker
(E) Operative correction of ptosis

A

The correct response is Option C.

In this 37-year-old woman who has symptomatic blepharoptosis following botulinum toxin injection, the most appropriate management is administration of alpha-adrenergic agonist eyedrops. Botulinum toxin (Botox) is associated with numerous other complications, including diplopia, retrobulbar hemorrhage, perforation of the globe, lagophthalmos, photophobia, epiphora, ectropion, and exposure keratitis. Ecchymoses, eyelid retraction, eyebrow ptosis, and asymmetry have also occurred in some patients.

Blepharoptosis results from migration of the toxin from the area of treatment (the glabellar region in this patient) into the levator muscle. This effect is typically transient, as the dose of migrated toxin diminishes significantly. Patients with mild to moderate symptoms should be reassured that condition is typically self-limiting, and no treatment is recommended. If the blepharoptosis is moderate to severe, ocular decongestants, such as the alpha-adrenergic agonists antazoline and naphazoline, are administered. These eyedrops act to contract Müeller’s muscle temporarily and thus elevate the upper eyelid margin, relieving the symptoms.

83
Q

Which of the following substances is NOT contained in Jessner’s solution?

(A) Ethanol
(B) Glycolic acid
(C) Lactic acid
(D) Resorcinol
(E) Salicylic acid

A

The correct response is Option B.

Jessner’s solution is incorporated into skin peeling agents to even the depth of the peel and improve exfoliation. This solution, which contains ethanol, lactic acid, resorcinol, and salicylic acid, is often used for treatment of hyperpigmentation. The mechanism of action of Jessner’s solution is believed to be destruction of intracellular connections between keratinocytes as well as removal of the epidermis. Its use results in increased epidermal turnover and a decreased quantity of melanin-counting keratinocytes.

Because the depth of penetration of Jessner’s solution is limited, the rate of complications is decreased. Mild erythema develops following the first application, which may worsen over time. Additionally, frosting of the skin may be seen with further application.

84
Q

A 55-year-old woman has had pain, swelling, and erythema of the left arm for the past 24 hours. She underwent mastectomy and axillary lymph node dissection on the left four years ago. On examination, she is afebrile. Laboratory studies show a leukocyte count that is within normal limits.

Which of the following is the most appropriate management?

(A) Lymphatic massage
(B) Application of a compression bandage and elevation of the extremity
(C) Topical application of an antibiotic
(D) Intravenous administration of an antibiotic
(E) Incision and drainage

A

The correct response is Option D.

In this patient who has had the spontaneous onset of cellulitis of the arm after undergoing axillary lymph node dissection, the most appropriate management is intravenous administration of an antistreptococcal antibiotic. Fever and leukocytosis are typically associated with cellulitis but are not required to make the diagnosis, as many of these patients will be afebrile and will not have an increased leukocyte count or absolute neutrophil count on serologic testing. Anti-streptolysin O titer may be positive.

Although lymphatic massage and compression and elevation of the extremity are useful in controlling the lymphedema associated with lymph node dissection, these measures will not treat cellulitis. Antibiotic therapy should not be based on the results of blood or tissue aspirate cultures because these often do not yield any growth. Topical application of an antibiotic will not effectively treat cellulitis. Incision and drainage of the affected site is not indicated.

85
Q

Which of the following best describes a patient with Fitzpatrick type II skin?

(A) Usually burns, tans with difficulty
(B) Sometimes burns, tans moderately
(C) Rarely burns, tans easily
(D) Never burns, deep pigmentation

A

The correct response is Option A.

Fitzpatrick’s system is used for classifying patients according to skin type in order to assist them in the promotion of skin care and the prevention of solar damage. The classification is also helpful in determining treatment protocols for topical and laser skin care. This method of classification is based on melanin content, inherent pigmentation, and sensitivity to ultraviolet light during an initial, unprotected sun exposure. Patients with a lower Fitzpatrick skin type tend to have less melanin pigmentation within their skin.

A table representing this classification system is shown below.

86
Q

A 55-year-old woman is scheduled to undergo 30% trichloroacetic acid peeling for eradication of fine perioral rhytids. Which of the following is the most likely complication?

(A) Cardiac arrhythmias
(B) Herpetic reactivation
(C) Hyperpigmentation
(D) Hypertrophic scarring
(E) Loss of sweat glands

A

The correct response is Option C.

Potential complications of trichloroacetic acid peeling are rare, but include infection, scarring, and changes in skin pigmentation. Pigmentary changes are most common. The optimal chemical peel creates a controlled partial-thickness wound; however, peels that destroy tissue through the entire epidermis are most likely to result in hypopigmentation. Hyperpigmentation, which is typically transient, results from inflammatory changes that are thought to be caused by trauma to melanocytes, resulting in excessive stimulation.

Cardiac arrhythmias are associated with phenol peeling, and herpetic reactivation is usually a complication of laser resurfacing. Trichloroacetic acid peeling provides a superficial peel that would not result in hypertrophic scarring or permanent loss of sweat glands.

87
Q

A 52-year-old woman wishes to undergo phenol chemical peeling for improvement of the facial skin surface. She has a history of alcohol abuse and liver disease. Laboratory evaluation shows a prothrombin time of 12 sec and a serum aspartate aminotransferase level of 68 U/L. Which of the following complications is most likely in this patient?

(A) Cardiac arrhythmias
(B) Delayed wound healing
(C) Excessive bleeding
(D) Hypertrophic scarring
(E) Permanent hyperpigmentation

A

The correct response is Option A.

Phenol (carbolic acid) is an aromatic hydrocarbon derived from coal tar. It is one of the most common agents used in chemical peeling and is often combined with other topical agents to increase absorption and decrease inflammatory skin response. However, patients who undergo phenol peeling should be monitored closely because of the potential for the development of cardiac toxicity associated with increased levels of phenol in the blood. Because phenol is detoxified in the liver, adverse cardiac effects are more likely to occur in this patient who has a history of alcohol abuse and liver disease. In addition, only small areas should be treated at one time; if chemical peeling is performed on more than one half of the face in less than 30 minutes, arrhythmias or other severe cardiac complications can occur. These complications have been noted in as many as 50% of treated patients.

Other, less common complications in patients undergoing phenol peeling include hypopigmentation, transient splotchy hyperpigmentation, prominence of the skin pores, telangiectasias, erythema, and milia. Delayed wound healing and hypertrophic scarring are rare complications of deep peeling. Bleeding does not occur.

88
Q

A 65-year-old woman desires correction of fine facial rhytids. Which of the following agents is most effective for skin rejuvenation in this patient?

(A) Ascorbic acid
(B) Glycolic acid
(C) Hyaluronic acid
(D) Retinoic acid
(E) Trichloroacetic acid

A

The correct response is Option E.

Because trichloroacetic acid produces moderate exfoliation in concentrations of 15% to 35%, it is the most appropriate agent for improvement of facial rhytids. The other agents listed provide only mild or no exfoliation.

Ascorbic acid (vitamin C) has been shown in some clinical studies to improve photoaged skin but is not yet widely used.

Glycolic acid and other alpha-hydroxy acids are used for chemical peeling. These agents provide some improvement of fine rhytids, but overall results in patients with photoaged skin remain controversial.

Hyaluronic acid is used in other countries as a filler substance for lip augmentation, as well as for correction of rhytids. It has also shown promise as an adjunct treatment for depressed scars. However, it is not yet approved for use in the United States.

Retinoic acid is also used for skin rejuvenation but has been shown to provide only moderate improvement in fine rhytids. Clear histologic improvement in photoaging has not been confirmed.

89
Q

Use of which of the following agents is CONTRAINDICATED prior to dermabrasion?

(A) Alpha-hydroxy acid
(B) Glycolic acid
(C) Hydroquinone 4%
(D) Isotretinoin
(E) Tretinoin

A

The correct response is Option D.

Isotretinoin (Accutane, also referred to as 13-cis retinoic acid) is contraindicated in a patient who is to undergo dermabrasion. Isotretinoin is an oral retinoid that is used to treat acne by suppressing keratinization and the function of sebaceous glands, thereby diminishing the oiliness of the skin. In patients undergoing isotretinoin therapy, dermabrasion or laser resurfacing should be deferred for at least one year following discontinuation of the drug, because delayed healing and hypertrophic scarring may occur.

Alpha-hydroxy and glycolic acids are mild agents typically found in many over-the-counter skin creams. These agents are not contraindicated in patients undergoing dermabrasion.

Hydroquinones are typically administered preoperatively and postoperatively to prevent hyperpigmentation.

Tretinoin is used for skin preparation in patients scheduled to undergo dermabrasion or laser therapy.

90
Q

A patient requests bovine collagen injection for correction of glabellar frown lines. Following administration of the required test dose, this patient should be observed for potential development of adverse effects for how long?

(A) 1 hour
(B) 1 day
(C) 1 week
(D) 1 month
(E) 1 year

A

The correct response is Option D.

Following injection of a test dose of bovine collagen (Zyderm/Zyplast), the patient should be observed for one month in order to note the development of any adverse effects. Clinical studies have reported hypersensitivity reactions in 3% of otherwise healthy persons who undergo skin testing. A hypersensitivity reaction is defined as the onset of erythema, induration, or swelling at the test site. Approximately 70% of these reactions occur within 72 hours, 10% occur within one week, and the remaining 20% occur within four weeks. If the patient has no signs of hypersensitivity at that time, the bovine collagen can then be injected into the glabellar frown lines.

It should be noted that 1% to 4% of patients who have nonreactive skin tests subsequently develop hypersensitivity reactions following treatment with injectable bovine collagen.

91
Q

A 39-year-old woman desires correction of deep frown lines between the eyebrows and at the bridge of the nose. Five injections of botulinum toxin (2.5 units per injection) are administered. Three days later, the patient says that she has not experienced improvement of the frown lines.

Which of the following is the most likely cause of the current findings?

(A) Excessively superficial level of injection
(B) Inactive toxin
(C) Inadequate dose of botulinum toxin
(D) Inadequate time for onset of action
(E) Prior immunity to botulinum toxin

A

The correct response is Option D.

Although botulinum toxin, an exotoxin of Clostridium botulinum bacteria, is currently only approved by the FDA for use in patients with strabismus and blepharospasm, it is increasingly being used for temporary eradication of facial wrinkling. Its neuromuscular mechanism of action occurs by preventing acetylcholine uptake. It must be reconstituted just prior to use, refrigerated, and then used within 48 hours (before the potency diminishes). Small doses (2.5 U) per site are injected directly into the muscle belly, with paralysis occurring within three to seven days and lasting for four to six months.

Because this patient has only waited three days since injection, the most likely explanation for the absence of paralysis at the treatment site is that the time for onset of action has been inadequate.

Complications resulting from botulinum toxin use are typically related to inadvertent diffusion of the toxin to the surrounding musculature. Potential diffusion can be minimized by using highly concentrated doses of the toxin, localizing the injection to the involved muscle, and instructing the patient to avoid any bending or straining following injection. Systemic and immunologic side effects may also be seen but are uncommon.

92
Q

Which of the following best describes the mechanism of action of retinoids on the skin?

(A) Decreased activation of metalloproteases resulting from inhibition of AP1 transcription
(B) Decreased free radical-mediated damage to the skin
(C) Increased desquamation resulting from diminished corneocyte cohesion
(D) Increased gene transcription of collagen types I and III
(E) Inhibition of the conversion of dopamine to melanin

A

The correct response is Option A.

The mechanism of action of retinoids involves decreased activation of metalloproteases through inhibition of AP1 transcription. Retinoic acid has been shown to reverse the effects of photoaging, while use of tretinoin results in thinning of the stratum corneum, thickening of the epidermis, reversal of atypia, and increasing collagen synthesis within the dermis, with angiogenesis and a more even dispersion of melanin granules. One report that studied the continuous use of retinoids for longer than four years showed improvement of rhytids and hyperpigmentation and increased skin smoothness. The mechanism of action of retinoids has been shown to occur through binding to a DNA receptor. The activated receptor then inhibits AP1 transcription factor, which inhibits the activation of such metalloproteases as collagen, gelatinase, and stromelysin.

Topical vitamin C is an experimental agent that has demonstrated promising results in limited studies. It has been shown to decrease the free radical-mediated effects of UVB radiation in mouse models, as well as to stimulate cultured fibroblasts, resulting in increased production of collagen types I and III through an increase in gene transcription. The mechanism of action of alpha-hydroxy acids, which gradually reduce fine rhytids, is thought to occur through increased desquamation resulting from diminished corneocyte cohesion immediately above the granular layer in the epithelium. Hydroquinones are commonly used bleaching agents that block the conversion of dopamine to melanin through inhibition of the tyrosinase enzyme.

93
Q

Dermabrasion is most appropriate for the treatment of which of the following conditions?

(A) Actinic keratoses on the cheeks
(B) Decorative tattoo on the upper back
(C) Hypertrophic burn scar of the shoulder
(D) Ice-pick acne scars
(E) Traumatic tattoo of the chin

A

The correct response is Option E.

Dermabrasion, which is a method of skin resurfacing involving abrasion of the epidermis, is especially useful for treatment of perioral rhytids and traumatic tattoos. Because traumatic tattooing involves the embedding of particulate matter within the superficial epidermis, dermabrasion can be used to uproot and remove the debris. Unlike chemical peeling and laser resurfacing, dermabrasion is an imprecise technique that can result in the development of complications if the abrasion is performed at a level deeper than the upper third of the dermis. These complications, which include hypertrophic scarring, hypopigmentation, and an enhanced, porous look to the skin, are more common in areas more distal to the head and neck because these regions have a decreased density of adnexal structures, which lie within the dermis and supply basal keratinocytes for skin resurfacing.

Options for treatment of sun-induced actinic damage of the cheeks include laser resurfacing, trichloroacetic acid peeling, and topical application of 5-fluorouracil. The use of dermabrasion for the removal of decorative tattoos is associated with a high incidence of postoperative scarring; again, a laser is the choice for removal. Dermabrasion will not effectively remove hypertrophic scars or keloids. Excision is preferred for management of ice-pick acne scars.

94
Q

The Fitzpatrick skin classification stratifies patients according to

(A) actinic skin damage and fine wrinkle formation
(B) the potential for pigmentary changes following chemical peeling
(C) their risk for cardiac toxicity associated with phenol peeling
(D) their risk for hypertrophic scarring following skin resurfacing
(E) thickness and laxity of facial skin

A

The correct response is Option B.

Fitzpatrick’s system is the most widely used method for classifying patients according to skin type in order to stratify their risk for the development of postinflammatory hyperpigmentation following chemical peeling and laser skin resurfacing. This method of classification is based on the patient’s skin pigmentation and subsequent response following exposure to ultraviolet light. A table representing this classification system is shown below.

Patients who have Fitzpatrick type I, type II, or type III skin have the lowest risk for development of hyperpigmentation following chemical peeling. In contrast, patients with type IV, type V, or type VI skin are at increased risk for pigmentary changes.

95
Q

A 42-year-old woman who desires correction of perioral and periorbital rhytids begins therapy with 0.025% tretinoin. Which of the following responses is most likely to be seen in this patient?

(A) Hypertrichosis
(B) Increased type III collagen
(C) Partial-thickness burn
(D) Subcutaneous atrophy
(E) Thinning of the dermis

A

The correct response is Option B.

Histologically, long-term application of tretinoin results in the formation of new type III embryonic collagen; this process is believed to be caused by activation of fibroblasts. This derivative of vitamin A, also known as retinoic acid, results in obliteration of atypia and microscopic actinic keratoses when applied once daily at a strength of 0.025% to 0.05%. Improvement of rhytids is not immediate, occurring instead over six to 12 months. During this time, the dermis becomes markedly thickened and the elasticity of the skin greatly increases; the latter is due to an increase in collagen levels of as much as 80%. Adverse effects are frequent and may include erythema, peeling, xerosis, pruritus, and the onset of a sunburn sensation.
Use of retinoic acid does not affect hair growth or loss. A partial-thickness burn is never associated with tretinoin therapy.

96
Q

A 56-year-old man who has pitted acne scars and ruddiness and oiliness of the skin surface. He desires improvement in the appearance of his skin. Which of the following is the most appropriate management?

(A) Alexandrite laser therapy
(B) Injection of bovine collagen
(C) Dermabrasion
(D) Oxygen therapy
(E) Punch excision

A

The correct response is Option C.

The most appropriate management of this patient’s acne scars is dermabrasion. This skin resurfacing technique, which can be used to improve acne scarring, is performed with a motorized instrument and diamond fraises. The appropriate depth of the dermabrasion is indicated by the onset of punctate bleeding; any deeper dermabrasion can result in hypertrophic scarring. However, even after treatment the scars may be perceptible, and patients who have deep “ice pick” acne scars, which are especially recalcitrant to treatment, may need to undergo multiple sessions or subsequent surgical excision.

Carbon dioxide laser resurfacing is an alternative to dermabrasion that can be used to improve either acne scarring,
Alexandrite lasers are used to remove hair or tattoos. Although bovine collagen can be injected to temporarily fill small acne scars, its effects generally last for only three to six months. In addition, as many as 3% of patients can experience a delayed hypersensitivity reaction. Oxygen therapy is a modality often used in spa-like settings. It cannot be used to improve acne scarring. Because punch excision is limited to small areas, it would not be effective for resurfacing this patient’s widespread acne scars.

97
Q

Which of the following topical agents provides effective protection against long-wave ultraviolet A, short-wave ultraviolet A, and ultraviolet B radiation?

(A) Octyl dimethyl paba (PABA)
(B) Octyl salicylate (OCS)
(C) Oxybenzone (Benzophenone-3)
(D) Titanium dioxide
(E) Zinc oxide

A

The correct response is Option E.

Zinc oxide is the only agent that has been shown to be effective against all ultraviolet A and ultraviolet B rays. While ultraviolet A rays are believed to contribute to elastic tissue damage and skin aging because of their constancy, ultraviolet B rays are thought to be the primary cause of most sunburns and acute skin damage due to their intensity during summer months. It is important to note that the SPF rating of a sunscreen measures its effectiveness against ultraviolet B radiation only. Because the overall effect of solar exposure on skin aging should instead be measured primarily by long-term exposure to both ultraviolet A and ultraviolet B radiation, it is important to use a sunscreen
Octyl dimethyl paba (PABA), octyl salicylate (OCS), and octyl methoxycinnamate (OMC) provide effective protection against ultraviolet B rays only. In contrast, oxybenzone (Benzophenone-3) protects against skin damage from short-wave ultraviolet A radiation. Titanium dioxide is effective against short-wave ultraviolet A as well as ultraviolet B radiation.

98
Q

In a 47-year-old woman who has used topical tretinoin for the last four years, which of the following histologic features is most likely?

(A) A compact stratum corneum
(B) A decrease in dermal mucin
(C) A decrease in epidermal thickness
(D) A decrease in hyaluronic acid
(E) An increase in melanin production

A

The correct response is Option A.

Topical tretinoin (Retin-A) can be used to decrease the effects of photoaging, including those resulting from solar exposure. Long-term studies have uncovered several histologic features resulting from the topical application of tretinoin, including compactness of the stratum corneum, which produces smoothing of the skin; an increase in hyaluronic acid, which is thought to reduce the appearance of fine lines; an increase in epidermal thickness with return of the granular layer thickness to a baseline value; an increase in dermal mucin; and a decrease in melanin production.

Adverse effects associated with tretinoin use include erythema and crusting of the skin. Alternate-day therapy or daily therapy using a dilute dose has been shown to decrease these effects. Because tretinoin is also associated with sun sensitivity, skin protection during sun exposure is advocated.